Вы находитесь на странице: 1из 163

UNIVERSITE DE LIEGE

FACULTE DES SCIENCES APPLIQUEES


DEPARTEMENT DELECTRICITE, ELECTRONIQUE ET
INFORMATIQUE
ELECTROMAGNETISME
Enonces des exercices du cours ELEN0076-1
septembre 2012
P. Rousseaux
B. Vanderheyden
Institut Monteore Bat. B28
Sart-Tilman, 4000 Li`ege.
1 Analyse vectorielle
1.1 Divergence et theor`eme de la divergence
Soit le vecteur

A = x x + y y + z z.
1. Calculez lintegrale
I
S
=
_
d

S

A (1)
sur la surface dune sph`ere de rayon R centree `a lorigine.
2. Calculez


A, ainsi que lintegrale
I
V
=
_
d
3
r
_


A
_
. (2)
sur le volume de la meme sph`ere.
3. Interpretez les resultats obtenus aux deux points precedents.
1.2 Rotationnel et theor`eme de Stokes
x
R
C
1
C
2
y
B A
Figure 1 exercice 1.2
Soit le vecteur

A = A
0
x o` u A
0
est une constante. Calculez les integrales
curvilignes suivantes :
I
1
=
_
C
1
d



A, (3)
I
2
=
_
C
2
d



A, (4)
o` u les chemins C
1
et C
2
sont representes `a la gure 1.
2
2 Electrostatique
2.1 Equations de base
Force de Coulomb entre deux charges ponctuelles : force exercee sur la
charge +q
2

F =
q
1
q
2
4r
2
r
+q
1
r

F
+q
2
(5)
Champ electrique produit par une charge ponctuelle :

E(P) =
q
1
4r
2
r
+q
1

E
P
r
(6)
Champ electrique produit par une distribution continue de charges sur
un volume V :
P
0

r
r

E(r) =
1
4
0
_
V
(r

)
|r r

|
3
_
r r

_
d
3
r

(7)
Deplacement electrique ou densite de ux :

D =

E
Equations de Maxwell (cas statique)


E = 0 (8)


D = . (9)
Potentiel electrique :


E = 0 =

E =

V .
Potentiel electrique relatif une distribution continue de charges sur un
volume V :
V (r) =
1
4
0
_
V
(r

)
|r r

|
d
3
r

(10)
3
Dierence de potentiel : V
B
V
A
=
_
B
A
d



E.
Energie emmagasinee sous forme electrique :
W
E
=
1
2
_
V
d
3
r

D

E. (11)
Capacite electrique dun syst`eme de deux conducteurs charges :
C =
Q
V
1
V
2
(12)
Theor`eme de Gauss
_
S
d

S

D = Q =
_
V
d
3
r (13)
Equation de Poisson :

2
V =

(14)
Equation de Laplace : si = 0,

2
V = 0 (15)
2.2 Dipole electrique
Un dipole electrique est constitue de deux charges electriques dampli-
tudes egales et de signes opposes, distantes de d et placees dans le vide
(gure 2).
1. Determinez lexpression du potentiel et du champ electrique du syst`eme
en un point r, dans la limite d |r|.
2. Representez les lignes de champ.
3. Determinez lexpression de la densite de charge electrique .
4
Figure 2 exercice 2.2
2.3 Distribution lineaire de charges
On consid`ere un syst`eme de charges electriques distribuees uniformement
sur une droite indenie, avec une densite lineique
L
(unites : C/m). Le
syst`eme est dans le vide (gure 3).
1. Determinez lexpression du champ electrique de cette distribution
(a) par integration directe,
(b) en utilisant le theor`eme de Gauss.
2. Determinez lexpression du potentiel electrique correspondant.
Figure 3 exercice 2.3
2.4 Capacite du condensateur plan - Equation de La-
place
On consid`ere un condensateur plan `a air dont les plateaux sont separes
dune distance d. Le potentiel du plateau superieur vaut V
0
volts, celui du
plateau inferieur vaut 0 volt.
5
1. En partant de lequation de Laplace, determinez les expressions du
potentiel et du champ electrique entre les deux plateaux. On negligera
les eets de bord.
2. Quelle est lexpression de la charge surfacique portee par le plateau
superieur ?
3. Determinez lexpression de la capacite de ce syst`eme.
2.5 Capacite dun cable coaxial
Un cable coaxial est constitue de deux conducteurs de meme axe et de
rayons respectifs a et b, avec b > a. Les conducteurs sont separes par un
milieu isolant de permittivite electrique .
1. Determinez lexpression de la capacite unitaire du syst`eme.
2. Donnez-en une valeur numerique dans le cas suivant : le rayon exterieur
est egal `a b = 0.5 cm, le conducteur interieur a une section de 1 mm
2
et la permittivite relative du dielectrique vaut
r
= 2.5.
2.6 Deux lignes chargees parall`eles
On consid`ere deux lignes uniformement chargees, portant respectivement
une charge lineique
L
et
L
, placees parall`element dans le vide `a une
distance d lune de lautre.
1. Determinez lexpression du potentiel electrique du syst`eme.
2. Esquissez lallure des courbes equipotentielles et des lignes de champ
electrique.
2.7 Distribution lineique de charges au dessus dun
plan conducteur : application de la methode des
images
Une ligne uniformement chargee (densite de charge
L
) est placee pa-
rall`element `a un plan conducteur dextension innie, `a une distance h de
celui-ci.
1. Determinez les expressions du potentiel V et du champ electrique

E au
dessus du plan conducteur.
2. Determinez la charge electrique induite par unite de surface dans le
plan conducteur.
3. A partir du principe de superposition, determinez les valeurs du champ
electrique aux points A et B du plan.
6
h
+
l
B
A
h
Figure 4 exercice 2.7
2.8 Ligne `a proximite dun conducteur cylindrique
On consid`ere une ligne uniformement chargee (charge lineique
L
) placee
parall`element `a un conducteur cylindrique plein de rayon a, porte `a un po-
tentiel nul (gure 5).
Determinez lexpression du potentiel electrique du syst`eme.
Figure 5 exercice 2.8
2.9 Capacite unitaire dune ligne bilaire
On consid`ere une ligne bilaire, constituee de deux conducteurs cylin-
driques de rayon a places parall`element `a une distance d lun de lautre.
En supposant que d a, etablissez lexpression de la capacite electrique
par unite de longueur du syst`eme.
2.10 Capacite dun syst`eme compose dun conducteur
cylindrique et dun plan conducteur
On consid`ere un cylindre conducteur place parall`element `a un plan conduc-
teur. Le cylindre est porte au potentiel V
0
, le plan conducteur est relie `a la
masse.
Determinez la capacite unitaire du syst`eme.
7
Figure 6 exercice 2.10
3 Polarisation electrique
3.1 Equations de base
Figure 7 exercice 3.1
Polarisation electrique

P = N p, o` u N est le nombre de molecules (ou
datomes) par unite de volume et p est le moment dipolaire moyen par
molecule (par atome).
Densite de charges liees en volume :

pol
=


P, (16)
Densite de charges liees en surface :

pol
= n

P. (17)
Dans un milieu lineaire, homog`ene et isotrope,

P =
0

E, (18)
o` u est la susceptibilite electrique.
Deplacement electrique, ou densite de ux electrique,

D =
0

E +

P =
0
(1 + )

E =
0

E (19)
8
+
pol,1 +

E

E
0
+
pol,2
Figure 8 exercice 3.2
3.2 Condensateur plan
On consid`ere un condensateur plan (`a dielectrique, de permittivite )
dont les plateaux ont une surface S et sont separes de d. Le condensateur est
soumis `a une dierence de potentiel de V
0
volts (gure 8).
1. Determinez les expressions de la densite de surface des charges libres, ,
de la densite de surface des charges liees,
pol
1,2
, et du champ electrique

E regnant entre les plateaux.


2. Deduisez-en lexpression de la capacite C du condensateur.
3. Calculez les valeurs numeriques de ,
pol
1,2
,

E et C dans les deux cas
suivants :
(a) S = 1 mm
2
, d = 1 mm, V
0
= 10 V et
r
= 1 (air),
(b) S = 1 mm
2
, d = 1 mm, V
0
= 10 V et
r
= 6 (mica).
3.3 Condensateur `a deux dielectriques
z

E
d = d
1
+ d
2
d
2

d
2
d
1
Figure 9 exercice 3.3
On consid`ere un condensateur plan dont la partie isolante est constituee
de deux couches dielectriques (gure 9). La premi`ere couche est de lair (
r
=
1) et a une epaisseur d
1
= 0.1 mm. La deuxi`eme couche est composee de
mica (
r
= 6) et a une epaisseur d
2
= 0.9 mm. La surface des plateaux vaut
S = 1 cm
2
et leur separation vaut d = 1 mm. Le condensateur est soumis `a
une dierence de potentiel constante, V
0
= 2.5 kV.
1. Determinez la valeur champ electrique dans chacune des couches de la
partie isolante. Le condensateur peut-il claquer ? (Champ de rupture
de lair : E
rupt
= 2.7 MV/m, champ de rupture du mica : E
rupt
=
14 MV/m).
9
2. Determinez la capacite du condensateur.
4 Magnetostatique
4.1 Equations de base
Loi de Biot-Savart :

H(r) =
1
4
_
V
d
3
r

J (r r

)
|r r

|
3
, (20)
ou, pour un courant I secoulant le long dun conducteur,

H(r) =
I
4
_
L
d

(r r

)
|r r

|
3
. (21)
Direction de

H : rappelez-vous la r`egle du tire-bouchon!
Induction magnetique, milieu de permeabilite :

B =

H.
Theor`eme de Gauss :


B = 0
_
S
d

S

B = 0 (S : surface fermee). (22)
Theor`eme dAmp`ere (en magnetostatique) :


H =

J
_

H d

=
_
S
d

S

J =

I. (23)
Potentiel vecteur :


B = 0 =

B =


A. (24)
Si

A est choisi de facon telle que


A = 0, alors

A(r) =

4
_
d
3
r

J(r

)
|r r

|
, (25)
ou, dans le cas dun courant I secoulant le long dun conducteur,

A(r) =
I
4
_
d

|r r

|
. (26)
Flux magnetique traversant une surface S :
=
_
S
d

S

B =
_
S
d

S
_


A
_
=
_

A d

. (27)
Energie emmagasinee sous forme magnetique :
W
M
=
1
2
_
V
d
3
r

H

B. (28)
Inductance :
= LI et W
M
=
1
2
LI
2
. (29)
10
4.2 Champ magnetique produit par une boucle de cou-
rant
On consid`ere une boucle de courant de rayon a parcourue par un courant
continu I.
Figure 10 exercice 4.2
Etablissez, par integration directe, lexpression du champ magnetique pro-
duit le long de laxe de la boucle.
4.3 Conducteur rectiligne indeni
On consid`ere un conducteur rectiligne inniment long, parcouru par un
courant continu I.
Etablissez lexpression du champ magnetique du syst`eme, des trois facons
suivantes :
1. en integrant la relation de Biot-Savart,
2. `a partir du potentiel vecteur

A,
3. en utilisant le theor`eme dAmp`ere.
4.4 Champ magnetique produit par un solenode `a air
Un solenode `a air est constitue dun l conducteur enroule en forme
de cylindre de rayon a et comporte n tours de l conducteur par m`etre de
longueur. La longueur du solenode est supposee beaucoup plus grande que
son rayon a. Determinez lexpression de linduction magnetique creee par ce
solenode en tout point interieur et exterieur eloigne de ses extremites. On
exploitera judicieusement les hypoth`eses et symetries du probl`eme.
4.5 Conducteur de section circulaire
On consid`ere un conducteur de rayon a parcouru par un courant continu
I.
11
Etablissez lexpression du champ magnetique
1. `a linterieur du conducteur,
2. `a lexterieur du conducteur.
4.6 Inductance dun cable coaxial
On consid`ere un cable coaxial de longueur innie alimente par un courant
continu I. Ses caracteristiques geometriques sont les suivantes :
rayon du conducteur interieur : a
rayon interieur du conducteur exterieur : b
rayon exterieur du conducteur exterieur : c
permeabilite du milieu isolant :
0
permeabilite du conducteur :
c
Determinez lexpression du champ magnetique produit dans les dierentes
regions du cable, ainsi que son inductance unitaire.
4.7 Inductance dune ligne bilaire
Figure 11 exercice 4.7
On consid`ere une ligne bilaire, cest-`a-dire une ligne constituee de deux
conducteurs rectilignes de rayon a separes de d et parcourus par des courants
continus egaux et opposes (gure 11).
Determinez linductance unitaire de la ligne.
5 Pertes - Champs variables - Phenom`enes
dinduction - Ondes
5.1 Reacteur chimique `a electrodes spheriques
Un reacteur chimique est constitue de deux electrodes spheriques concen-
triques de rayons respectifs a = 1 cm et b = 3 cm. Le reacteur est rempli
dun liquide de permittivite relative
r
= 78 et de conductivite = 3 S/m.
En regime, le reacteur est parcouru par un courant continu.
Determinez la resistance electrique de ce reacteur.
12
5.2 Condensateur avec pertes
Determinez la conductance dun condensateur plan au mica aux frequences
f
1
= 50 Hz et f
2
= 1 GHz. Les caracteristiques du condensateur sont les sui-
vantes
Surface des plateaux : S = 10 cm
2
Separation des plateaux : d = 0.1 cm
Conductivite du mica : = 10
15
S/m
Proprietes dielectriques `a f = 50 Hz :

= 6
0
et pertes dielectriques negligeables
Proprietes dielectriques `a f = 1 GHz :

= 6
0
et

tot
= 1.6 10
3

0
.
5.3 Blindage electrique
Un ecran metallique est place entre les deux plateaux dun condensateur
`a air, comme illustre `a la gure 12.
Figure 12 exercice 5.3
Le syst`eme est alimente par un generateur de tension sinusodale (ampli-
tude de tension V
0
, frequence f). Au moment du montage et avant la mise
en service du generateur, lecran est non-charge. La surface des plateaux est
egale `a S.
Determinez :
1. les courants

I
1
et

I
2
,
2. lallure des lignes de champ electrique,
3. la distribution des charges sur lecran,
dans chacun des deux cas suivants :
1. lecran est `a un potentiel ottant (il est isole galvaniquement des autres
elements du syst`eme),
2. lecran est relie `a la masse.
13
5.4 Pertes dun bobinage : variation en fonction de la
frequence
On consid`ere un bobinage `a noyau ferromagnetique. Comment varient les
pertes suivantes en fonction de la frequence ?
1. Les pertes par hysteresis.
2. Les pertes dues aux courants de Foucault.
Dans chaque cas, on exprimera la puissance dissipee P sous la forme P f
n
,
o` u n est la puissance algebrique correspondante.
5.5 Puissance solaire recue par Mercure
La puissance du rayonnement solaire frappant la surface de Mercure vaut
approximativement S = 0.87 W/cm
2
.
1. En supposant que les ondes electromagnetiques sont emises par le soleil
de facon isotrope, estimez la puissance totale rayonnee par le soleil.
2. En assimilant le rayonnement `a la surface de mercure `a une onde plane,
estimez la valeur ecace du champ electrique produit par ce rayonne-
ment.
Donnee : distance Mercure - Soleil, d = 60 Gm.
5.6 Absorption dune onde plane par une plaque de
cuivre
Une onde plane de frequence f = 1 GHz se propage dans lair et frappe
une plaque de cuivre, `a incidence normale (voir gure 13).
S
r
E
i
H
i
S
i
z
y
x S
t
H
t
E
t
H
r
E
r
Figure 13 exercice 5.6
Sachant que la valeur de crete du champ electrique de londe incidente
vaut E
0
= 1 V/m et que la conductivite electrique du cuivre vaut =
58 10
6
S/m, determinez la valeur moyenne de la puissance absorbee par unite
de surface par le cuivre.
14
5.7 Application du theor`eme de Poynting `a un bloc
conducteur
On consid`ere un bloc de cuivre semi-inni setendant dans lespace z > 0,
comme illustre `a la gure 14, et une onde plane incidente venant frapper le
bloc `a incidence normale.
Figure 14 exercice 5.7
Etablissez lexpression de la puissance dissipee dans le bloc de cuivre `a
partir du vecteur de Poynting de londe transmise dans le bloc. Exprimez
ensuite cette puissance en fonction de lamplitude du champ magnetique `a la
surface du bloc et veriez que le resultat obtenu est identique `a celui etabli
au cours par integration directe de la puissance dissipee par eet Joule.
15
6 Lignes de transmission en regime harmo-
nique
6.1 Param`etres unitaires dune ligne telephonique
Une ligne telephonique de type bilaire, representee `a la gure 15, est
constituee de deux conducteurs cylindriques de rayon a = 0.2 mm places pa-
rall`element lun `a lautre `a une distance d = 0.6 mm. Les ls sont isoles
electriquement par un plastique (
r
= 2). Les ls sont en cuivre ( =
5.8 10
7
S/m,
r
= 1 et
r
= 1).
Figure 15 exercice 6.1
A la frequence de travail f = 1 kHz, on a mesure les quatre param`etres
unitaires suivants : R = 100 /km, C = 0.051 F/km, L = 0.6 mH/km et
G 0 S/m.
1. Cette ligne travaille-t-elle en regime de haute ou de basse frequence ?
2. Travaille-t-elle en regime de faibles pertes ?
3. Determinez limpedance caracteristique, lattenuation lineique et la vi-
tesse de phase.
6.2 Param`etres unitaires dune ligne microbande
La ligne microbande (microstrip en anglais) de la gure 16 est constituee
dun substrat isolant et dielectrique, metallise sur ses deux faces avec de
laluminium. La face superieure vehicule un courant allant du generateur `a
la charge, le retour seectuant par la face inferieure.
Determinez les quatre param`etres unitaires C, L, R et G, limpedance
caracteristique Z
0
et le coecient dattenuation de cette ligne.
Donnees :
Frequence de travail : f = 18 GHz.
Aluminium : = 3.72 10
7
S/m,
r
= 1 et
r
= 1.
Dielectrique : Si0
2
, et pertes dielectriques negligeables,
r
= 3.8 et
r
= 1.
16
Figure 16 exercice 6.2
6.3 Param`etres unitaires dun cable coaxial
Un cable coaxial, represente `a la gure 17, presente les param`etres sui-
vants :
Param`etres geometriques : a = 1 mm, b = 1 cm et d = 1 mm.
Conducteurs : cuivre, = 5.8 10
7
S/m, =
0
et =
0
.
Dielectrique : polystyr`ene, = 10
16
S/m, =
0
, et

= 2.5
0
,

pol
/

= 10
5
`a f = 1 kHz ;

pol
/

= 2 10
4
`a f = 1 GHz
Figure 17 exercice 6.3
On demande de determiner les param`etres unitaires C, L, R et G, limpedance
caracteristique Z
0
et le coecient dattenuation de ce cable, aux deux frequences
f = 1 kHz et f = 1 GHz.
Interpretez les dierences observees.
6.4 Allure de lamplitude de tension le long dune ligne
en regime harmonique
Une ligne de transmission ideale dimpedance caracteristique Z
0
= 75
et de longueur = 0.6 est terminee sur une charge dimpedance Z
L
=
100 + j150 .
1. Determinez le taux dondes stationnaires S et la position des maxima
et minima de la tension le long de la ligne.
2. Esquissez lallure de |

V (z)|.
17
3. Determinez limpedance vue au droit de lemplacement z = 0.4,
limpedance dentree de la ligne ainsi que les impedances Z
min
et Z
max
vues au droit des minima et maxima de tension.
6.5 Mesure dune impedance en hautes frequences
Une ligne ideale dimpedance caracteristique Z
0
= 50 alimente une
impedance de charge Z
L
, que lon cherche `a determiner. La mesure de la
tension le long de la ligne fournit un taux dondes stationnaires S = 3. Le
premier minimum de tension est egalement localise `a une distance z
min
=
0.33 de la charge.
Determinez la valeur de limpedance de charge Z
L
.
6.6 Ligne ideale quart donde
On consid`ere un troncon de ligne ideale dimpedance caracteristique Z
0
et dune longueur egale au quart de la longueur donde du signal transmis.
Pour chacune des situations suivantes :
1. ligne laissee ouverte `a son extremite ;
2. ligne court-circuitee ;
3. ligne terminee sur une resistance de charge R
1
< Z
0
;
4. ligne terminee sur une resistance de charge R
2
> Z
0
;
determinez :
le coecient de reexion et le taux dondes stationnaires ;
lexpression et lallure du module de la tension et du courant le long de
la ligne ;
limpedance vue de lentree de la ligne.
6.7 Adaptation par troncon de ligne quart donde
Une ligne de longueur
1
= 10 m et dimpedance caracteristique Z
01
=
300 doit etre connectee `a une deuxi`eme ligne, dimpedance caracteristique
Z
02
= 150 , de longueur
2
= 20 m, et terminee sur une charge Z
L
= 150 .
Les lignes sont constituees de cables coaxiaux remplis avec un dielectrique
de permittivite relative
r
= 2.25 et sont considerees sans perte. La frequence
de travail est f = 50 MHz.
1. Si les deux lignes sont connectees directement, determinez le taux
dondes stationnaires dans le troncon de longueur
1
.
Z
01
= 300 Z
02
= 150

1
Z
L
18
2. On souhaite `a present connecter les deux lignes par lintermediaire dun
adaptateur quart donde, constitue dun cable coaxial rempli avec le
meme dielectrique.
Z
0
Z
01
= 300 Z
02
= 150

1
4

2
Z
L
adaptateur
(a) Determinez la longueur et limpedance caracteristique Z
0
du
troncon intermediaire permettant de realiser ladaptation `a la premi`ere
ligne.
(b) Determinez le coecient de reexion au droit de lentree de la
deuxi`eme ligne (impedance caracteristique Z
02
) et au droit de
lentree de ladaptateur. En deduire le taux dondes stationnaires
dans le troncon de longueur
1
et celui dans ladaptateur.
6.8 Adaptation dun reseau dantennes par ajout dun
stub
Un reseau dantennes est constitue de la mise en parall`ele dun ensemble
de 10 antennes identiques, chacune alimentee par une ligne ideale comme
indique `a la gure 18. Chaque antenne est adaptee `a sa ligne dalimentation
de sorte que limpedance caracteristique de chaque ligne dalimentation Z
0
est
egale `a limpedance de lantenne Z
L
= 50 . On desire realiser ladaptation
de ce reseau dantennes `a sa ligne dalimentation principale dimpedance
caracteristique Z
0
= 50 . A cette n, on place en parall`ele sur la ligne
dalimentation principale, `a une distance
1
du reseau dantennes, un stub en
court-circuit de meme impedance caracteristique Z
0
et de longueur
2
.
Determinez les valeurs de
1
et
2
qui realisent ladaptation. Pour ces
valeurs, que vaut le taux dondes stationnaires (i) dans la ligne au droit du
reseau dantennes, (ii) dans le stub au droit du court-circuit ?
Donnees :
Frequence de travail : f = 1 GHz.
Constantes caracteristiques des lignes :
r
= 2.3 et
r
= 1.
6.9 Bilan de puissance dans un quart donde en regime
harmonique
Un generateur de tension ecace V
g
= 15 V et dimpedance interne Z
g
=
75 alimente une charge dimpedance Z
L
= 60 par lintermediaire dune
ligne de transmission ideale dimpedance caracteristique Z
0
= 75 et de
longueur = /4 (voir gure 19).
19
Figure 18 exercice 6.8
Figure 19 exercice 6.9
1. Que vaut limpedance dentree Z
in
() ?
2. Que vaut la puissance delivree `a la ligne ?
3. Quelle est la tension developpee aux bornes de la charge, en z = 0 ?
4. Quelle est la puissance dissipee par la charge ?
5. Esquissez lallure de |

V (z)| pour < z < 0.


6.10 Adaptation de ligne et adaptation conjuguee
On consid`ere le montage de la gure 20, pour lequel on souhaite comparer
plusieurs types dadaptation.
Etant donne limpedance dentree Z
in
= R
in
+jX
in
et limpedance interne
du generateur, Z
g
= R
g
+ jX
g
,
1. etablissez lexpression de la puissance delivree `a la charge ;
20
Figure 20 exercice 6.10
2. parmi les deux schemas dadaptation suivants : Z
L
= Z
0
(adaptation
de ligne) et Z
g
= Z
in
(adaptation de generateur), quel est celui qui
garantit la plus grande puissance delivree `a Z
L
?
3. Determinez la valeur de Z
in
pour laquelle la puissance delivree `a la
charge est maximale (adaptation conjuguee).
7 Lignes de transmission en regime transi-
toire
7.1 Propagation dune impulsion carree dans une ligne
ideale
+

V
g
1 ns
1 V
V
g
R
g
R
L
Z
0
ligne ideale
0
z
t
Figure 21 exercice 7.1
Une ligne ideale dimpedance caracteristique Z
0
= 50 , de longueur
= 2 m et de vitesse de phase v
p
= 10
8
m/s est terminee sur une resistance
de charge R
L
= 20 , et est alimentee par un generateur de resistance interne
R
g
= 30 . Ce generateur delivre un signal sous forme dimpulsions carrees
de duree = 1 ns et de tension V
0
= 1 V, comme illustre `a la gure 21.
On demande
1. de tracer lallure de la tension et du courant en fonction du temps,
au droit de z = 0 et z = /2. On se limitera aux deux premi`eres
impulsions recues `a ces positions ;
2. de calculer la fraction de lenergie fournie au generateur qui est dissipee
dans la charge R
L
.
21
7.2 Propagation dun echelon de tension dans une ligne
ideale
La ligne ideale de la gure 22 a pour param`etres Z
0
= 50 , = 2 m et
v
p
= 2 10
8
m/s.
Elle est terminee sur une resistance de charge R
L
= Z
0
/2, et alimentee
par un generateur de resistance interne R
g
= Z
0
/2. Ce generateur delivre un
signal echelon damplitude 1 V.
+

0
z
V
g
t = 0
R
g
= Z
0
/2
R
L
= Z
0
/2
Z
0
ligne ideale
Figure 22 exercice 7.2
Esquissez lallure de la tension en z = en fonction du temps durant
les 50 premi`eres millisecondes apr`es letablissement de lechelon de tension.
7.3 Charge dun condensateur en regime transitoire
Un generateur alimente un condensateur de capacite C par lintermediaire
dune ligne de transmission ideale. En supposant que linterrupteur est ferme
en t = 0 et que le condensateur est initialement non charge, determinez
lallure de la tension V (z, t) le long de la ligne.
+

ligne ideale
0
z
C
R
g
= Z
0
V
0
t = 0
Z
0
Figure 23 exercice 7.3
22
8 Guides dondes
8.1 Propagation dun mode TEM dans un guide dondes
coaxial
Le cable coaxial de la gure 24 a les param`etres suivants :
Figure 24 exercice 8.1
rayon interieur : a = 1.2 mm
rayon exterieur : b = 3.25 mm
epaisseur du conducteur exterieur : d = 2 mm
proprietes dielectriques `a f = 100 MHz :

r
= 1.3 et

r
0
conducteurs consideres parfaits.
Le cable est alimente par un signal sinusodal damplitude V
0
= 500 V et
de frequence f = 100 MHz. Ce signal se propage sous la forme dun mode
TEM (onde progressive).
1. Esquisser lallure des lignes de champ de

E et

H dans une section
transverse.
2. Determiner les expressions des phaseurs

E et

H en fonction de V
0
et
des param`etres du cable. Travailler en coordonnees cylindriques.
3. Determiner lexpression et la valeur numerique de la puissance moyenne
transportee par le signal, (i), en utilisant le vecteur de Poynting, (ii),
en utilisant la theorie des lignes de transmission (pour rappel, Z
0
=
(/2)ln[b/a]).
23
8.2 Modes TM des guides dondes rectangulaires
On consid`ere le guide dondes metallique de section rectangulaire de la
gure 25. Ses parois sont des conducteurs parfaits, linterieur est rempli dun
isolant de permittivite electrique et de permeabilite magnetique .
Figure 25 exercice 8.2
Determinez les expressions de la composante

E
z
du champ electrique
correspondant aux modes TM, cest-`a-dire les modes tels que

H
z
= 0.
Remarques :
Travaillez dans le domaine frequentiel et limitez-vous `a des solutions
dondes progressives, du type

E
z
(x, y, z) =

E
0z
e
z
, o` u est la constante
de propagation (complexe).
Une fois

E
z
(x, y, z) determine, il vous sura dutiliser les relations en
rotationnel pour obtenir les autres composantes :

E = j

H


E
z
y



E
y
z
= j

H
x
(30)


E
x
z



E
z
x
= j

H
y
(31)


E
y
x



E
x
y
= j

H
z
(32)

H = j

E


H
z
y



H
y
z
= j

E
x
(33)


H
x
z



H
z
x
= j

E
y
(34)


H
y
x



H
x
y
= j

E
z
(35)
8.3 Cavite resonante parallelepipedique
On consid`ere un guide dondes de section rectangulaire, court-circuite `a
ses deux extremites (en z = 0 et en z = d) par des conducteurs plans parfaits.
Lorsque quun mode TE
10
se propage dans ce guide, il subit des reexions
aux deux plans, ce qui se traduit par la formation dondes stationnaires.
24
Figure 26 exercice 8.3
1. Determinez lexpression de la frequence des modes stationnaires qui se
developpent dans la cavite, ainsi que lexpression de la composante y
du champ electrique correspondant.
2. Dans le cas dune cavite `a air de forme cubique (1 cm de cote), quelle
est la plus petite frequence de resonance ?
Donnees : mode TE
10
,

E
y
= E
0
sin
_
x
a
_
e
jz
, (36)
o` u =
_

2

2
/a
2
.
8.4 Mode dun guide dondes de section rectangulaire
On consid`ere un guide dondes de section rectangulaire, avec a = 1.5 cm
et b = 0.8 cm, et dont lespace entre les conducteurs est rempli par un
dielectrique de caracteristiques = 0 ,
r
= 1 et
r
= 4.
On rel`eve
H
x
= 2 sin
_
x
a
_
cos
_
3y
b
_
cos
_
10
11
t z
_
[A/m] . (37)
Determiner
1. le mode en operation,
2. la frequence de coupure correspondante,
3. le vecteur donde ,
4. la constante de propagation .
8.5 Perturbation des modes dun guide dondes de sec-
tion rectangulaire
On ins`ere une plaque metallique depaisseur negligeable dans le plan
median dun guide donde rectangulaire `a air (la plaque est positionnee en
y = b/2).
25
Pour lesquels des modes suivants la repartition des champs electromagnetiques
est-elle aectee par linsertion de la plaque ?
modes TE
20
, TE
01
, TM
11
, TM
12
, TE
11
.
Justiez.
8.6 Nombre de modes permis en dessous dune frequence
donnee
On consid`ere un guide dondes de section rectangulaire (a = 2.5 cm et
b = 1 cm) `a dielectrique ( = 0,
r
= 1 et
r
= 4) pour une application o` u
lon travaille au maximum `a la frequence f = 15.1 GHz.
Combien de modes TE et TM ce guide peut-il transmettre ?
8.7 Frequence de coupure dun guide dondes de sec-
tion rectangulaire
On mesure la longueur donde du mode fondamental dun guide dondes
rectangulaire `a air travaillant `a f = 15 GHz.
Si cette longueur donde est egale `a 2.5 cm, que vaut la frequence de
coupure de ce mode et quelles sont les dimensions possibles du guide ?
8.8 Puissance transportee dans un guide dondes de
section rectangulaire
On consid`ere un guide dondes `a air, de section rectangulaire (a = 3 cm
et b = 1.5 cm), vehiculant son mode fondamental et travaillant `a la frequence
f = 1.1 f
c10
. Sachant que le champ electrique est de la forme

E
y
= E
0
sin
_
x
a
_
e
jz
, (38)
o` u E
0
= 4 10
4
V/m, calculez la puissance moyenne transportee par ce mode.
8.9 Temps de parcours
Un guide rectangulaire `a air (a = 2b = 2.5 cm) a une longueur de 100 m.
Le guide est alimente `a une de ses extremites par un signal consistant en une
porteuse de 10 GHz modulee par impulsions.
1. Determinez quels modes propageront ce signal.
2. Quel est le temps de parcours dune impulsion dans le guide ?
26
9 Antennes
9.1 Rayonnement dun dipole de Hertz
On souhaite estimer lordre de grandeur de la puissance electromagnetique
absorbee par lutilisateur dun GSM.
Pour ce faire, on modelise lantenne du telephone par un dipole de Hertz
travaillant `a la frequence f = 900 MHz et on calcule la puissance rayonnee
dans un plan place `a 5 cm de lantenne.
Figure 27 exercice 9.1
Si la puissance totale demission est egale `a 0.1 W, quelle est la repartition
de la puissance rayonnee au travers du plan?
9.2 Principe des antennes images
Comment peut-on modeliser, `a laide du principe des images, le rayonne-
ment dun dipole de Hertz place au dessus dun plan conducteur connecte `a
la masse ?
9.3 Antennes reseau
On consid`ere deux dipoles de Hertz places parall`element `a une distance
/4 et alimentes par des courants dephases de /2 (

I
2
=

I
1
e
j/2
).
Determiner le diagramme de rayonnement correspondant dans le plan
equatorial ( = /2).
Figure 28 exercice 9.3
27
UNIVERSITE DE LIEGE
FACULTE DES SCIENCES APPLIQUEES
DEPARTEMENT DELECTRICITE, ELECTRONIQUE ET
INFORMATIQUE
ELECTROMAGNETISME
Seances dexercices - transparents
septembre 2012
P. Rousseaux
Institut Monteore Bat. B28
Sart-Tilman, 4000 Li`ege.
TP1 :
RAPPEL ANALYSE VECTORIELLE
ELECTROSTATIQUE -

E et V
ELEN0076 1 - 1
vecteur :

A = A
x
x +A
y
y +A
z
z

A
x
y
z
produit scalaire :

A

B = A
x
B
x
+A
y
B
y
+A
z
B
z
= [

A[[

B[ cos

A

produit vectoriel :

A

B = (A
y
B
z
A
z
B
y
) x + (A
z
B
x
A
x
B
z
) y
+ (A
x
B
y
A
y
B
x
) z = [

A[[

B[ sin n
=

x y z
A
x
A
y
A
z
B
x
B
y
B
z

A
ELEN0076 1 - 2
Op erateur

:

=

x
x +

y
y +

z
z
uniquement en coordonn ees cart esiennes
Gradient (dune grandeur scalaire) :

A =
A
x
x +
A
y
y +
A
z
z
Laplacien (dune grandeur scalaire) :

2
=

2
x
2
+

2
y
2
+

2
z
2
(=


)
ELEN0076 1 - 3
Divergence (dune grandeur vectorielle) :


A =
A
x
x
+
A
y
y
+
A
z
z
Rotationnel (dune grandeur vectorielle) :


A =(
A
z
y

A
y
z
) x + (
A
x
z

A
z
x
) y + (
A
y
x

A
x
y
) z
=

x y z

z
A
x
A
y
A
z

ELEN0076 1 - 4
Coordonn ees cylindriques :
r =
_
x
2
+y
2
x = r cos
=arctan
y
x
y = r sin
dx , dy , dz dr , rd , dz
x
z
r

r
z
y
Coordonn ees sph eriques :
r =
_
x
2
+y
2
+z
2
x = r sin cos
=arctan
_
x
2
+y
2
z
y = r sin sin
=arctan
y
x
z = r cos
dx , dy , dz dr , rd , r sin d
y

r
r

x
z
ELEN0076 1 - 5
ELEN0076 1 - 6
Th eor` eme de la divergence
_
V
(


A) d
3
r =
_
S

A

dS
V
S
1.1 Divergence et th eor` eme de la divergence
Soit le vecteur

A = x x +y y +z z.
1. Calculez lint egrale
I
S
=
_
S
d

S

A
sur la surface dune sph` ere de rayon R centr ee ` a lorigine.
2. Calculez


A, ainsi que lint egrale
I
V
=
_
V
d
3
r
_


A
_
.
sur le volume de la m eme sph` ere.
ELEN0076 1 - 7
Sch ema de r esolution :
1. Coordonn ees sph eriques
sur la sph` ere

A = R r

dS = R
2
sin dd r
I
S
=
_

0
_
2
0
R
3
sin dd = 4R
3
2.


A =
A
x
x
+
A
y
y
+
A
z
z
=
1
r
2

r
(r
2
A
r
) = 3
I
V
=
_
R
0
_

0
_
2
0
3r
2
sin drdd = 4R
3
ELEN0076 1 - 8
Th eor` eme de Stokes
_
S
(


A)

dS =
_
C

A

d
c
S
1.2 Rotationnel et th eor` eme de Stokes
Soit le vecteur

A = A
0
x o` u A
0
est une con-
stante. Calculez les int egrales curvilignes sui-
vantes :
I
1
=
_
C
1
d



A,
I
2
=
_
C
2
d



A.
x
c
1
R
c
2
B A
y
ELEN0076 1 - 9
Sch ema de r esolution :
1. sur c
1
:

d = R[d[(

)

= sin x + cos y
_
C
1

A

d =
_
B
A
A
0
Rsin (d)
=
_

0
A
0
Rsind = 2A
0
R
2. sur c
2
:

d = dxx
_
C
2

A

d =
_
B
A
A
0
dx
=
_
R
R
A
0
dx = 2A
0
R
3.

A est un champ irrotationnel :


A = 0
ELEN0076 1 - 10
Champ electrique cr e e (dans le vide)
par une charge ponctuelle q

E =
q
4
0
r
2
r

E
+q
r
P
par une ensemble de charges ponctuelles : application de la superposition

E =

i
q
i
4
0
r
2
i
r
i
par une charge distribu ee sur un volume V

E(r) =
1
4
0
_
V
(r

)
[r r

[
3
_
r r

_
d
3
r

0
r
P
r

ELEN0076 1 - 11
Equations de Maxwell relatives ` a

E, r egime statique


E =


E = 0
permittivit e di electrique du vide :
0
=
1
36
10
9
= 8.85 10
12
F/m
Potentiel electrique V


E = 0 =

E =

V
V d etermin e ` a une constante pr ` es.
Diff erence de potentiel entre deux points A et B
V
A
V
B
=
_
A
B

E

d =
_
B
A

E

d
Diff erence de potentiel ind ependante du chemin choisi !
ELEN0076 1 - 12
Potentiel electrique cr e e (dans le vide)
par une charge ponctuelle q
V =
q
4
0
r
+C
par une ensemble de charges ponctuelles : application de la superposition
V =

i
q
i
4
0
r
i
+C
par une charge distribu ee sur un volume V
V (r) =
1
4
0
_
V
(r

)
[r r

[
d
3
r

+C

0
r
P
r

ELEN0076 1 - 13
2.2 Dip ole electrique
Un dip ole electrique est constitu e de deux charges
electriques damplitudes egales et de signes op-
pos es, distantes de d et plac ees dans le vide.
1. D eterminez lexpression du potentiel et du
champ electrique du syst ` eme en un point r,
dans la limite d [r[.
2. Repr esentez les lignes de champ.
3. D eterminez lexpression de la densit e de
charge electrique .
ELEN0076 1 - 14
1.
Coordonn ees sph eriques
Par superposition
V (r) =
q
4
0
1
[ r
1
[

q
4
0
1
[ r
2
[
r
1
= r
d
2
z r
2
= r +
d
2
z
[r
1
[ =
_
r
2
+
d
2
4
rd cos [r
2
[ =
_
r
2
+
d
2
4
+rd cos
Pour d r
1
_
r
2
+
d
2
4
rd cos

1
r

1
2
d cos
r
2
V =
qd cos
4
0
1
r
2
=
p
4
0
z r
r
2
avec p = qd le moment dipolaire
ELEN0076 1 - 15
Champ electrique

E =

V =
V
r
r
1
r
V

=
p
4
0
_
2 cos r + sin

r
3
_
2. Lignes de champ et equipotentielles
Equation des equipotentielles :
p cos
4
0
1
r
2
= V
0
=r =
_
p cos
4
0
V
0
_1
2
Lignes de champ en tout point aux
equipotentielles
ELEN0076 1 - 16
3. Charge volumique
=
0


E =
1
r
2

r
(r
2
E
r
) +
1
r sin

(sin E

)
= 0
Finalement = 0 OUF !
ELEN0076 1 - 17
2.3 Distribution lin eaire de charges
On consid` ere un syst ` eme de charges electriques
distribu ees uniform ement sur une droite ind enie,
avec une densit e lin eique
L
(unit es : C/m).
1. D eterminez lexpression du champ electrique
de cette distribution
(a) par int egration directe,
(b) en utilisant le th eor ` eme de Gauss.
2. D eterminez lexpression du potentiel
electrique correspondant.
ELEN0076 1 - 18
1. Calcul du champ

E
Par int egration directe

E(r) =
1
4
0
_

L
(r

)(r r

)
[r r

[
3
dz

r = r r r

= z

z [r r

[ =
_
r
2
+z

2
E
z
= 0 E
r
=

L
4
0
_

r
(

r
2
+z

2
)
3
dz

Changement de variable z

= r tan
E
r
=

L
4
0
_
/2
/2
r
cos
3

r
3
r
cos
2

d =

L
2
0
r
r
z
z

r r

r
r

L
dz

ELEN0076 1 - 19
Th eor` eme de Gauss
Application du th eor ` eme de la divergence ` a l equation de Maxwell


E =

0
_
S

E

dS =
_
V


E d
3
r =
_
V

0
d
3
r

S
V
En pratique :
exploiter la ou les sym etries du probl ` eme
choisir de mani ` ere judicieuse la surface de Gauss S
ELEN0076 1 - 20
Par application du th eor ` eme de Gauss
Surface de Gauss = cylindre de hauteur h et de rayon r
Par sym etrie

E = E
r
(r) r
_
S

E

dS =
_
S
1

E

dS +
_
S
2

E

dS
. .
=0
+
_
S
3

E

dS =
_
h
0

L
dz

0
E
r
2rh =

L
h

0
E
r
=

L
2
0
r
S
1
h
r

E
S
2
S
3
ELEN0076 1 - 21
2. Calcul du potentiel V

E =

V =
V
r
r
V =

L
2
0
ln r +C
V =

L
2
0
ln
r
0
r
+C
C permet de xer la valeur de V pour la dimension caract eristique r
0
3. Lignes de champ et equipotentielles
Equation des equipotentielles :

L
2
0
ln
r
0
r
= V
0
r = r
0
e
2
0
V
0

L
ELEN0076 1 - 22
Equipotentielles = cylindres de rayon r
Lignes de champ

E radiales
ELEN0076 1 - 23
TP2 : ELECTROSTATIQUE
EQUATION DE LAPLACE - CAPACITE
METHODE DES IMAGES
ELEN0076 2 - 1
R esolution de probl ` emes d electrostatique via l equation de Laplace
Le potentiel V satisfait ` a l equation de Poisson

2
V =

0
Dans une r egion de lespace d epourvue de charges sources ( = 0), equation de
Laplace:

2
V = 0
M ethode de calcul de V dans un domaine de lespace ext erieur aux charges sources:
rechercher une solution ` a
2
V = 0 dans le domaine de lespace consid er e;
exploiter les propri et es de sym etrie pour deviner une solution;
appliquer les conditions aux limites sur les fronti ` eres du domaine.
Th eor` eme dunicit e
La solution ` a ce probl ` eme est unique !
ELEN0076 2 - 2
Condensateur et capacit e
Condensateur :
ensemble de deux conducteurs portant des charges egales et oppos ees +Q, Q,
faisant natre une diff erence de potentiel V entre les deux conducteurs.
+Q

E
Q
Capacit e du condensateur :
C =
Q
V
C est une constante d ependant de la g eom etrie du syst ` eme (ainsi que du mat eriau
s eparant les deux conducteurs)
ELEN0076 2 - 3
D etermination de la capacit e dun condensateur.
Deux proc edures possibles :
1. Imposer la d.d.p V entre les deux conducteurs
rechercher lexpression du potentiel V et du champ electrique dans lespace
entre les deux conducteurs ( equation de Laplace)
en d eduire la charge Qport ee par les conducteurs (th eor ` eme de Gauss)
2. Imposer la charge Qport ee par les conducteurs
rechercher lexpression du champ electrique dans lespace entre les deux
conducteurs (th eor ` eme de Gauss)
en d eduire la d.d.p. V entre les deux conducteurs.
ELEN0076 2 - 4
2.4 Capacit e du condensateur plan - Equation de Laplace
On consid` ere un condensateur plan ` a air dont les plateaux sont s epar es dune distance d.
Le potentiel du plateau sup erieur vaut V
0
volts, celui du plateau inf erieur vaut 0 volt.
1. En partant de l equation de Laplace, d eterminez les expressions du potentiel et du
champ electrique entre les deux plateaux. On n egligera les effets de bord.
2. Quelle est lexpression de la charge surfacique port ee par le plateau sup erieur ?
3. D eterminez lexpression de la capacit e de ce syst ` eme.
ELEN0076 2 - 5
1. Calcul du potentiel V
z
V
0
Q
+Q

E
d
Effets de bord n eglig es :

E = E
z
(z) z, V = V (z)
Domaine de lespace : 0 z d
Conditions aux limites :
V = 0 pour z = 0
V = V
0
pour z = d
Solution g en erale de
2
V :

2
V
z
2
= 0 V (z) = Az +B
Application des conditions aux limites :
V = 0 pour z = 0 : B = 0
V = V
0
pour z = d : A =
V
0
d
V (z) = V
0
z
d


E =
V
0
d
z
ELEN0076 2 - 6
2. Charge surfacique:


E est nul en tout point ext erieur au condensateur
Application du th eor ` eme de Gauss sur un parall elipip` ede de base S 0 et
entourant la plaque sup erieure du condensateur

0
E S = Q
S
,
0
E =
Q
S
S
lim
S0
(
Q
S
S
) =
S

0
E =
S

S
=
0
V
0
d
3. Capacit e
Q =
S
S =
0
V
0
S
d
C =
Q
V
0
=

0
S
d
ELEN0076 2 - 7
2.5 Capacit e dun c able coaxial
Un c able coaxial est constitu e de deux conducteurs de m eme axe et de rayons respectifs
a et b, avec b > a. Les conducteurs sont s epar es par un milieu isolant de permittivit e
electrique .
1. D eterminez lexpression de la capacit e unitaire du syst ` eme.
2. Donnez-en une valeur num erique dans le cas suivant : le rayon ext erieur est egal ` a
b = 0.5 cm, le conducteur int erieur a une section de 1 mm
2
et la permittivit e relative
du di electrique vaut
r
= 2.5.
ELEN0076 2 - 8
1. Capacit e du syst ` eme
Recherche du potentiel V dans lespace s eparant les deux conducteurs
R esolution de l equation de Laplace

2
V = 0 pour a r b avec V = V (r) et
_
V = V
0
pour r = a
V = 0 pour r = b
Coordonn ees cylindriques
1
r

r
(r
V
r
) = 0
V
r
=
A
r
V = Aln r +B
r = a : V
0
= Aln a +B
r = 0 : 0 = Aln b +B
V =
V
0
ln a/b
ln
r
b

E =

V =
V
r
r =
V
0
lnb/a
1
r
r
ELEN0076 2 - 9
D etermination de la densit e de charge lin eique
L
port ee par le cylindre int erieur
Application du th eor ` eme de Gauss sur la surface dun cylindre de rayon a r b
et de hauteur h = 1m
2rE
r
=
L
E
r
=

L
2r
Expression identique ` a celle du conducteur
lin eique !
Remplacant E
r
= f(V
0
) :

L
=
2V
0
ln b/a
b

L
a
r
C =

L
V
0
=
2
lnb/a
2. Num eriquement :
C = 63.7 pF/m
ELEN0076 2 - 10
Lignes de champ et equipotentielles

E
+
+

E = 0
-
-
-
-
-
-
-
-
-
-
-
-
-
-
-
-
+
+
+
+
+
+
+
+
+

E = 0
ELEN0076 2 - 11
2.6 Deux lignes charg ees parall ` eles
On consid` ere deux lignes uniform ement charg ees, portant respectivement une charge
lin eique
L
et
L
, plac ees parall ` element dans le vide ` a une distance d lune de lautre.
1. D eterminez lexpression du potentiel electrique du syst ` eme.
2. Esquissez lallure des courbes equipotentielles et des lignes de champ electrique.
z
d
r

r
+

L
P(0, y, z)
x
y
ELEN0076 2 - 12
1. Expression du potentiel V
Application de la superposition
V (x, y, z) = V
+
+V

=

L
2
0
ln
[r

[
[r
+
[
+C
=

L
2
0
ln
_
y
2
+ (z +
d
2
)
2
_
y
2
+ (z
d
2
)
2
+C
Si C = 0 : V 0 pour P
2. Equation des equipotentielles
Donn ees par :
[r

[
[r
+
[
=
_
y
2
+ (z +
d
2
)
2
_
y
2
+ (z
d
2
)
2
= a
ELEN0076 2 - 13
V = 0
V < 0
V > 0
y
z
y
2
+
_
z
d
2
a
2
+ 1
a
2
1
_
2
= (
da
a
2
1
)
2
Equation de cercles de rayon
da
|a
2
1|
et de centre (0,
d
2
a
2
+1
a
2
1
)
ELEN0076 2 - 14
2.7 Distribution lin eique de charges au dessus dun plan conducteur : application
de la m ethode des images
Une ligne uniform ement charg ee (densit e de charge
L
) est plac ee parall ` element ` a un
plan conducteur dextension innie, ` a une distance h de celui-ci.
1. D eterminez les expressions du potentiel V et du champ electrique

E au dessus du
plan conducteur.
2. D eterminez la charge electrique induite par unit e de surface dans le plan conducteur.
3. A partir du principe de superposition, d eterminez les valeurs du champ electrique aux
points A et B du plan.
B
+
L
V = 0
z
y
h
h
A
ELEN0076 2 - 15
Probl ` eme :
R esoudre
2
V = 0 dans le domaine z 0
avec les conditions limites
_
V 0 pour y, z
V = 0 pour z = 0
En labsence de plan conducteur
V =

L
2
0
ln
r
0
_
y
2
+ (z h)
2
satisfait
2
V = 0 mais pas la condition limite V = 0 ( equipotentielles = cercles
concentriques ` a
L
)
Par contre, le potentiel
V =

l
2
0
ln
_
y
2
+ (z +h)
2
_
y
2
+ (z h)
2
satisfait ` a l equation de Laplace et aux conditions limites dans le domaine z 0
En vertu du th eor ` eme dunicit e, cest la solution du probl ` eme !
ELEN0076 2 - 16
M ethode des images

L
y
z
V = 0
+
L
d = 2h
Le potentiel solution est equivalent ` a celui cr e e par
deux charges lin eiques parall ` eles s epar ees dune
distance d = 2h.
Le plan conducteur est remplac e par une charge
L
, image de la charge
L
par
rapport au plan, de mani ` ere ` a satisfaire les conditions limites sur le plan.
ELEN0076 2 - 17
Champ electrique :

E =

V =
V
y
y
V
z
z
=

L
2
0
_
y y + (z h) z
y
2
+ (z h)
2

y y + (z +h) z
y
2
+ (z +h)
2
_
Densit e de charge surfacique sur le plan conducteur :

S
= n
0

E[
z=0
=
0
[

E[
z=0
=

L

h
y
2
+h
2
On observe :
_

S
(y)dy =
L
!
V = 0
n

E[
z=0

L
ELEN0076 2 - 18
Lignes de champ :
V = 0
y
z
ELEN0076 2 - 19
Champ

E aux points A et B
Par superposition des champs cr e es par la charge
L
et son image
L

L
B A

E
1

E
2

E
2

E
1
+
L
en A :

E
1
du ` a
L
:

L
2
0
h
( z)

E
2
du ` a
L
:

L
2
0
h
( z)

E =

E
1
+

E
2
=

L

0
h
z
en B :

E
1
du ` a
L
:

L
2
0
( y z)
2h

E
2
du ` a
L
:

L
2
0
( y z)
2h

E =

E
1
+

E
2
=

L
2
0
h
z
ELEN0076 2 - 20
TP3 : ELECTROSTATIQUE
METHODE DES IMAGES - POLARISATION
ELEN0076 3 - 1
2.8 Ligne ` a proximit e dun conducteur cylindrique
On consid` ere une ligne uniform ement charg ee (charge lin eique
L
) plac ee parall ` element
` a un conducteur cylindrique plein de rayon a, port e ` a un potentiel nul.
D eterminez lexpression du potentiel electrique du syst ` eme.
V = 0
a

L
d
ELEN0076 3 - 2
Probl ` eme :
R esoudre
2
V = 0 dans le domaine r a
avec les conditions limites
_
V Cte pour r
V = 0 pour r = a
Application de la m ethode des images
placer une charge lin eique image
L
de facon ` a obtenir une equipotentielle
V = 0 pour r = a, surface du cylindre conducteur
lemplacement de la charge image d

doit permettre de satisfaire cette condition


ELEN0076 3 - 3
a
0
d

L
d


L
P
r
r
+
r

V = 0
A B
Forme g en erale du potentiel :
V (r) =

L
2
0
ln
[r

[
[r
+
[
+C
Conditions en r = a :
au point A :

L
2
0
ln
d

+a
d+a
+C = 0
au point B :

L
2
0
ln
ad

da
+C = 0
_

_
d

=
a
2
d
C =

L
2
0
ln
d
a
ELEN0076 3 - 4
Solution du probl ` eme en coordonn ees polaires :
V =
_

L
2
0
ln

r
2
d
2
+a
4
2ra
2
d cos

r
2
a
2
+a
2
d
2
2ra
2
d cos
pour r a
0 pour r < a
Lignes de champ et equipotentielles :

L
ELEN0076 3 - 5
Autres g eom etries qui pr esentent la m eme conguration image:
Deux cylindres parall ` eles

L
Charge lin eique ` a lint erieur dun cylindre creux
Remarque :
Si d a on a d

a : on peut supposer la charge image plac ee au centre du


conducteur cylindrique.
ELEN0076 3 - 6
2.9 Capacit e unitaire dune ligne bilaire
On consid` ere une ligne bilaire, constitu ee de deux conducteurs cylindriques de rayon a
plac es parall ` element ` a une distance d lun de lautre.
En supposant que d a, etablissez lexpression de la capacit e electrique par unit e de
longueur du syst ` eme.
ELEN0076 3 - 7
Soient
L
et
L
les densit es de charges port ees par les deux conducteurs
M ethode des images : remplacer les conducteurs par deux charges lin eiques ` a
placer ` a lint erieur des deux conducteurs
Hypoth` ese : d a les charges images sont suppos ees plac ees au centre de
chaque conducteur
a

L
B
A
d
a
Potentiel du conducteur
L
evalu e au point A :
V
+
=

L
2
0
ln
d
a
+C
Potentiel du conducteur
L
evalu e au point
B :
V

=

L
2
0
ln
a
d
+C
Diff erence de potentiel entre les deux conducteurs :
V = V
+
V

=

L

0
ln
d
a
C =

L
V
=

0
lnd/a
ELEN0076 3 - 8
2.10 Capacit e dun syst ` eme compos e dun conducteur cylindrique et dun plan
conducteur
On consid` ere un cylindre conducteur plac e parall ` element ` a un plan conducteur. Le
cylindre est port e au potentiel V
0
, le plan conducteur est reli e ` a la masse.
D eterminez la capacit e unitaire du syst ` eme.
ELEN0076 3 - 9
Recherche du potentiel. Probl ` eme :
R esoudre
2
V = 0 dans le domaine r a, z > 0
avec les conditions limites
_
V = 0 pour z = 0
V = V
0
pour r = a
Application de la m ethode des images :
Le cylindre conducteur et le sol sont remplac es par deux charges image lin eiques
portant les densit es de charges
L
et
L
et plac ees ` a lint erieur des conducteurs
ELEN0076 3 - 10
a
A
B
h
d

L
d
Forme g en erale du potentiel :
V =

L
2
0
ln
r

r
+
+C
Il faut :
V (z = 0) = 0 h d

=
dd

2
et
C = 0
V (A) = V (B) = V
0

L
2
0
ln
d a
a d

=

L
2
0
ln
d +a
a +d

= a
2
/d
V
0
=

L
2
0
ln
d
a
=

L
2
0
ln
h +

h
2
a
2
a
C =
2
0
ln
h+

h
2
a
2
a
ELEN0076 3 - 11
Polarisation electrique
Dans un milieu di electrique, sous lapplication dun champ electrique :
des dip oles el ementaires se cr eent et/ou sorganisent pour sorienter dans le sens du
champ electrique appliqu e = polarisation
apparition de charges li ees distribu ees en volume
pol
et de charges li ees
distribu ees en surface
pol
: charges li ees ou de polarisation
Soit

P le moment dipolaire par unit e de volume, on a :

pol
=

pol
=

P n
ELEN0076 3 - 12
Vecteur d eplacement electrique

D :

D =
0

E +

P


D =

(
0

E +

P) = (
libres
+
pol
)
pol
=
libres
Dans un milieu lin eaire, homog` ene et isotrope

P =
0

D =
0
(1 +)

E =
0

E =

E
est la susceptibilit e electrique du milieu

r
est la permittivit e relative du milieu
est la permittivit e electrique du milieu
ELEN0076 3 - 13
3.2 Condensateur plan

+
+
pol,1
+
pol,2

On consid` ere un condensateur plan ( ` a di electrique, de permittivit e ) dont les plateaux


ont une surface S et sont s epar es de d. Le condensateur est soumis ` a une diff erence de
potentiel de V
0
volts.
1. D eterminez les expressions de la densit e de surface des charges libres, , de la
densit e de surface des charges li ees,
pol
1,2
, et du champ electrique

E r egnant
entre les plateaux.
2. D eduisez-en lexpression de la capacit e C du condensateur.
3. Calculez les valeurs num eriques de ,
pol
1,2
,

E et C dans les deux cas suivants :
(a) S = 1 mm
2
, d = 1 mm, V
0
= 10 V et
r
= 1 (air),
(b) S = 1 mm
2
, d = 1 mm, V
0
= 10 V et
r
= 6 (mica).
ELEN0076 3 - 14
Expression du potentiel et du champ electrique dans le condensateur: voir exercice
2.4 (effets de bord n eglig es)
V =
V
0
d
z

E =
V
0
d
z
charges li ees induites par polarisation
milieu lin eaire homog` ene et isotrope :

P =
0

E
charges li ees induites en volume
pol
=


P = 0
charges li ees induites en surface

+
+

+
+

+
+

+
+

+
+

+
+

+
+

+
+

+
+

libres
+
pol,2

libres
+
pol,1

pol,1

libres

libres

pol,2

pol
1
=

P n = P =
0
E
pol
2
=

P n = P =
0
E
ELEN0076 3 - 15
A la surface de la plaque sup erieure :

0
E =
libres
+
pol,1

libres
=
0
( 1 +)E = E =
V
0
d
Pour V
0
(ou E) x e, la pr esence du di electrique permet daugmenter les charges
libres stock ees sur les plateaux du condensateur
Pour
libres
x e, la pr esence du di electrique permet de r eduire la champ electrique
n ecessaire
Capacit e du condensateur :
C =
Q
libres
V
0
=
S
libres
V
0
=
S
d
ELEN0076 3 - 16
Valeurs num eriques :
E = 10
4
V/m
1. Condensateur ` a air :
Polarisation n egligeable

libres
=
0
E = 8.85 10
8
C/m
2

pol,1
=
pol,2
= 0
C = 8.85 10
3
pF
2. Condensateur ` a di electrique de permittivit e
Charges libres et charges li ees induites par polarisation

libres
= E = 53.12 10
8
C/m
2

pol,1
=
pol,2
= 44.27 10
8
C/m
2
C = 53.12 10
3
pF
ELEN0076 3 - 17
3.3 Condensateur ` a deux di electriques
d
1
= d
1
+ d
2
z

E
d
2

d
2
On consid` ere un condensateur plan dont la
partie isolante est constitu ee de deux couches
di electriques. La premi ` ere couche est de lair
(
r
= 1) et a une epaisseur d
1
= 0.1 mm. La
deuxi ` eme couche est compos ee de mica (
r
= 6)
et a une epaisseur d
2
= 0.9 mm. La surface des
plateaux vaut S = 1 cm
2
et leur s eparation vaut
d = 1 mm. Le condensateur est soumis ` a une
diff erence de potentiel constante, V
0
= 2.5 kV.
1. D eterminez la valeur champ electrique dans chacune des couches de la partie
isolante. Le condensateur peut-il claquer ? (Champ de rupture de lair :
E
rupt
= 2.7 MV/m, champ de rupture du mica : E
rupt
= 14 MV/m).
2. D eterminez la capacit e du condensateur.
ELEN0076 3 - 18
Q
Q

E
2

E
2

E
1

E
1

E = 0

E = 0

0
ELEN0076 3 - 19
Soit Q, Qla charge port ee par les deux armatures du condensateur
Champ electrique

E = E(z) z
dans lair (d
2
< z < d) : application du th eor ` eme de Gauss autour de larmature
1 (surface de Gauss = parall elipip` ede de base S) :
S
0
E
1
= Q

E
1
=
Q
S
0
z
dans le di electrique (0 < z < d
2
) : application du th eor ` eme de Gauss autour de
larmature 2 :
SE
2
= Q

E
2
=
Q
S
z
V erication : application du th eor ` eme de Gauss autour de linterface entre lair et
le mat eriau di electrique
S
0
E
1
+SE
2
= 0
pas de charge libre sur la surface de s eparation des deux milieux
ELEN0076 3 - 20
Diff erence de potentiel V
0
:
V
0
=
_
1
2

E

d =
_
d
2
0
E
2
dz +
_
d
1
+d
2
d
2
E
1
dz = Q(
d
2
S
+
d
1

0
S
)
Capacit e du syst ` eme :
C =
1
d
2
S
+
d
1

0
S
=
1
1
C
2
+
1
C
1
Expression de E
1
et E
2
en fonction de V
0
E
1
=
V
0
d
1
1
d
2
d
(1

0

)
E
2
=
V
0
d
1
1
d
1
d
(1

0
)
ELEN0076 3 - 21
z z
E
d
d
2
d
d
2
V
Valeurs num eriques :
E
1
= 10 000 kV/m E
2
= 1667 kV/m C = 3.54 pF
Claquage ?
E
1
> E
ruptair
la couche dair claque
V
0
appliqu e ` a la couche de di electrique d
2
E

2
=
V
0
d
2
= 2778 kV/m < E
ruptdie
C

=
S
d
2
= 5.57 pF
ELEN0076 3 - 22
TP4 : MAGNETOSTATIQUE
ELEN0076 4 - 1
Loi de Bio-Savart
Induction magn etique produite par un conducteur parcouru par un courant I continu
(dans le vide) :

B
0
r
P
r

Id

I

B(r) =

0
4
_
Id

(r r

)
[r r

[
3
Pour une distribution volumique de courant

J(r)
0

J
r
P
r

B(r) =

0
4
_

J(r

) (r r

)
[r r

[
3
d
3
r

ELEN0076 4 - 2
Champ magn etique dans le vide :

H =

0
= 4 10
7
H/m : perm eabilit e magn etique du vide
Champ magn etique dans un mat eriau :

H =

r
H/m : perm eabilit e magn etique du mat eriau
Equations de Maxwell relatives ` a

B et

H, r egime statique


B = 0 (pas de charge magn etique)


H =

J
ELEN0076 4 - 3
4.2 Champ magn etique produit par une boucle de courant
On consid` ere une boucle de courant de rayon a parcourue par un courant continu I.
Etablissez, par int egration directe, lexpression du champ magn etique produit le long de
laxe de la boucle.
Sch ema de r esolution
Coordonn ees cylindriques
Application de la loi de Bio-Savart :

r
r

r
I
I

d
r r

x
z
y

H =
I
4
_

d
r r

[r r

[
3
r = z z r

= a r

d = a d

H
r
= 0 H
z
=
a
2
I
2(a
2
+z
2
)
3/2
ELEN0076 4 - 4
Lignes de champ magn etique
ELEN0076 4 - 5
4.3 Conducteur rectiligne ind eni
On consid` ere un conducteur rectiligne inniment long, parcouru par un courant continu I.
Etablissez lexpression du champ magn etique du syst ` eme, des trois facons suivantes :
1. en int egrant la relation de Biot-Savart,
2. ` a partir du potentiel vecteur

A,
3. en utilisant le th eor ` eme dAmp` ere.
ELEN0076 4 - 6
1. Application de la loi de Bio-Savart

H =
I
4
_

d
(r r

)
[r r

[
3
r = r r r

= z z [r r

[ =
_
r
2
+z
2
I

d = I dz z
H

=
I
4
_

r
(

r
2
+z
2
)
3
dz =
I
2r
voir exercice 2.2 pour le calcul de lint egrale

H =
I
2r

z
I

d
r

r
r
r r

z
ELEN0076 4 - 7
Potentiel vecteur

A


B = 0

B =

A est d eni ` a une fonction scalaire pr ` es.


Si on impose


A = 0, on d erive une equation similaire ` a l equation de Poisson :

2

A =
0

J
Expression du potentiel vecteur pour une distribution de courant

J

A(r) =

0
4
_

J(r

)
[r r

[
d
3
r

ELEN0076 4 - 8
Th eor` eme dAmp` ere
Application du th eor ` eme de Stokes ` a l equation de Maxwell


H =

J
_
S
(


H)

dS =
_
C

H d

=
_
S

J

dS
En pratique :
exploiter la ou les sym etries du probl ` eme
choisir de mani ` ere judicieuse le contour c
ELEN0076 4 - 9
2. A partir du potentiel vecteur

A =

0
4
_

Idz

r
2
+z
2
_
divergente
calculer

A pour un troncon [L, L] en un point P du plan m edian z = 0

A =

0
4
_
L
L
Idz

r
2
+z
2
z
=

0
4
2
_
L
0
Idz

r
2
+z
2
z
=

0
I
2
ln
_
L
r
+
_
(
L
r
)
2
+ 1
_
z
Pour
L
r
on a

A

0
I
2
ln
2L
r
z
z
I dz z
L
L
r

r
r
r r

z
ELEN0076 4 - 10
Induction

B

B =


A =
A
z
r

=

0
I
2r



H =
I
2r

3. Application du th eor ` eme dAmp` ere : contour = cercle de rayon r


Par sym etrie :

H = H(r)

I =
_
C

H

d = 2rH(r)
H(r) =
I
2r
S

H
c
I
r
ELEN0076 4 - 11
4.4 Champ magn etique produit par un sol enode ` a air
Un sol enode ` a air est constitu e dun l conducteur enroul e en forme de cylindre de rayon
a et comporte n tours de l conducteur par m` etre de longueur. La longueur du
sol enode est suppos ee beaucoup plus grande que son rayon a. D eterminez lexpression
de linduction magn etique cr e ee par ce sol enode en tout point int erieur et ext erieur
eloign e de ses extr emit es. On exploitera judicieusement les hypoth` eses et sym etries du
probl ` eme.
ELEN0076 4 - 12
S
3
r
S
1
S
2
ELEN0076 4 - 13
Effets de bord n eglig es, sym etrie cylindrique : gure (a)

H = H
r
(r) r +H

(r)

+H
z
(r) z
montrons que H
r
= 0 : par int egration de


B = 0 sur le volume dun cylindre
concentrique
_
V


Bd
3
r = 0 =
_
S

B

dS
_
S

H d

S =
_
S
1

0
H
z
dS
_
S
2

0
H
z
dS
. .
=0
+
_
S
3

0
H
r
dS
=
0
2rH
r
= 0 H
r
= 0 r
car H
z
ind ependant de z
ELEN0076 4 - 14
montrons que H

= 0 r : gure (b)
application du th eor ` eme dAmp` ere
pour r < a
_
C
1

H

d =
_
S
1

J

dS = 0
2rH

= 0 H

= 0
pour r > a
_
C
2

H

d =
_
S
2

J

dS = 0
2rH

= 0 H

= 0
On suppose que les lignes de courant sont parfaitement circulaires
ELEN0076 4 - 15

ELEN0076 4 - 16
D etermination de H
z
par application du th eor ` eme dAmp` ere : gure (c)
pour r > a
_
DCC

H

d = H
z
(r
2
) H
z
(r
3
) = 0
H
z
(r
2
) = H
z
(r
3
) = Cte r
2
, r
3
> a
Or H
z
= 0 pour r H
z
= 0 r > a
pour r < a
_
ABCD

H

d = H
z
(r
1
) H
z
(r
2
) = nI
H
z
(r
2
) = 0 H
z
(r
1
) = nI r
1
< a
ELEN0076 4 - 17
4.5 Conducteur de section circulaire
On consid` ere un conducteur de rayon a parcouru par un courant continu I.
Etablissez lexpression du champ magn etique
1. ` a lint erieur du conducteur,
2. ` a lext erieur du conducteur.

J
z
ELEN0076 4 - 18
Effets de bord n eglig es :

H = H(r)

Application du th eor ` eme dAmp` ere sur un cercle de rayon r


` a lint erieur du conducteur r < a
_
C

H

d =
_
S

J

dS
densit e de courant :

J =
I
a
2
z
a

J

H r
2rH(r) = I
r
2
a
2
H(r) =
I
2
r
a
2

H =
I
2
r
a
2


B =

c
I
2
r
a
2

ELEN0076 4 - 19
` a lext erieur du conducteur r > a
a

H
r
2rH(r) = I
H(r) =
I
2r

H =
I
2r


B =

0
I
2r

ELEN0076 4 - 20
Flux dinduction magn etique
Flux dinduction magn etique ` a travers une surface S:

S
=
_
S

B

dS
Flux embrass e par une boucle de courant : ux dinduction magn etique ` a travers la
surface limit ee par la boucle de courant

dS

B
I
=
S
Pour un circuit ` a N spires (parcourues par un m eme
courant I):
= N
S
ELEN0076 4 - 21
Inductance
Inductance du circuit : (syst ` eme lin eaire)
L =

I
D enition alternative de linductance : syst ` eme capable de stocker une energie
magn etique
Energie magn etique stock ee dans un volume V
W
M
=
1
2
_
V

B

Hd
3
r =
1
2
LI
2
L =
2W
M
I
2
ELEN0076 4 - 22
4.6 Inductance dun c able coaxial
On consid` ere un c able coaxial de longueur innie aliment e par un courant continu I. Ses
caract eristiques g eom etriques sont les suivantes :
rayon du conducteur int erieur : a
rayon int erieur du conducteur ext erieur : b
rayon ext erieur du conducteur ext erieur : c
perm eabilit e du milieu isolant :
0
perm eabilit e du conducteur :
c
D eterminez lexpression du champ magn etique produit dans les diff erentes r egions du
c able, ainsi que son inductance unitaire.
ELEN0076 4 - 23
Expression du champ magn etique par application du th eor ` eme dAmp` ere : contour =
cercle de rayon r
_
C

H

d =
_
S

J

dS
densit e de courant :

J
int
=
I
a
2
z,

J
ext
=
I
(c
2
b
2
)
z
` a lint erieur du conduteur int erieur : r < a

J
int

H r
a

H
1
=
I
2
r
a
2


B
1
=

c
I
2
r
a
2

voir exercice 4.5


ELEN0076 4 - 24
entre les deux conducteurs : a < r < b

J
int
a
r

H
2
=
I
2r


B
2
=

0
I
2r

voir exercice 4.5


` a lint erieur du conducteur ext erieur: b < r < c

J
int

J
ext
b

H
r
2rH(r) = I(1
(r
2
b
2
)
(c
2
b
2
)
)

H
3
=
I
2r
(1
r
2
b
2
c
2
b
2
)


B
3
=

c
I
2r
(1
r
2
b
2
c
2
b
2
)

` a lext erieur du conducteur ext erieur : r > c

H =

B = 0 Blindage !
ELEN0076 4 - 25
Evolution de

H en fonction de r
Inductance : compos ee de trois parties
L
cint
: inductance li ee au champ magn etique pr esent dans le conducteur
int erieur
L
cext
: inductance li ee au champ magn etique pr esent dans le conducteur
ext erieur
L
i
: inductance li ee au champ magn etique pr esent dans lespace compris entre
les deux conducteurs
ELEN0076 4 - 26
D etermination de L
i
: recherche du ux total embrass e par la spire de courant (pour
1 m de longueur)
S
I
I

H
=
_
S

B
2


dS =

0
I
2
_
b
a
dr
r
=

0
I
2
ln
b
a
L
i
=

I
=

0
2
ln
b
a
H/m
ELEN0076 4 - 27
D etermination de L
cint
: recherche de l energie magn etique emmagasin ee dans 1 m
de conducteur
W
M
=
1
2
_
V

B
1


H
1
d
3
r =

c
I
2
16
L
cint
=
2W
M
I
2
=

c
8
H/m
D etermination de L
cext
: recherche de l energie magn etique emmagasin ee dans 1 m
de conducteur
W
M
=
1
2
_
V

B
3


H
3
d
3
r =

c
I
2
4
_
c
4
(c
2
b
2
)
2
ln
b
c

1
4
b
2

3
4
c
2
c
2
b
2
_
L
cext
=

c
2
_
c
4
(c
2
b
2
)
2
ln
b
c

1
4
b
2

3
4
c
2
c
2
b
2
_
H/m
ELEN0076 4 - 28
4.7 Inductance dune ligne bilaire
On consid` ere une ligne bilaire, cest- ` a-dire une ligne constitu ee de deux conducteurs
rectilignes de rayon a s epar es de d et parcourus par des courants continus egaux et
oppos es.
D eterminez linductance unitaire de la ligne.
Sch ema de r esolution
L = L
i
+L
c1
+L
c2
L
c1
= L
c2
=

c
8
ELEN0076 4 - 29
L
i
d etermin e ` a partir du calcul du ux total embrass e par la boucle de courant (1m
de longueur)


H est suppos e nul en dehors de lespace s eparant les deux conducteurs
dans lespace entre les deux conducteurs :
H(x) =
I
2x
+
I
2(d x)
I I
H

H
+
a
d
x
I
I

H
=

0
I
2
_
da
a
(
1
x
+
1
d x
)dx =

0
I

ln
d a
a
L
i
=

0

ln
d a
a
H/m
ELEN0076 4 - 30
TP5 : CONDUCTANCE - PERTES
CHAMPS VARIABLES
ELEN0076 5 - 1
Conductance dun condensateur
En r egime statique : li ee au courant de conduction

J =

E
est la conductivit e du mat eriau di electrique qui s epare les deux conducteurs.
Par d enition, pour une diff erence de potentiel V et un courant total I traversant le
mat eriau di electrique:
Q
V
c
S

J

E
+Q
I = G
c
V G
c
=
I
V
=
_
S

J

dS
_
C

E

d
S = surface ferm ee qui entoure un des conducteurs
c = chemin qui joint les deux conducteurs
On observe
G
c
C
=

avec C la capacit e du condensateur


Pertes associ ees ` a G
c
= pertes Joule
ELEN0076 5 - 2
M ethode de calcul : similaire ` a celle dune capacit e
1. Imposer la d.d.p V entre les deux conducteurs
rechercher lexpression du potentiel V et du champ electrique dans lespace
entre les deux conducteurs ( equation de Laplace)
en d eduire le courant I traversant le condensateur
2. Imposer le courant I
rechercher lexpression du champ electrique dans lespace entre les deux
conducteurs
en d eduire la d.d.p. V entre les deux conducteurs.
ELEN0076 5 - 3
5.1 R eacteur chimique ` a electrodes sph eriques
Un r eacteur chimique est constitu e de deux electrodes sph eriques concentriques de
rayons respectifs a = 1 cm et b = 3 cm. Le r eacteur est rempli dun liquide de
permittivit e relative
r
= 78 et de conductivit e = 3 S/m. En r egime, le r eacteur est
parcouru par un courant continu.
D eterminez la r esistance electrique de ce r eacteur.
ELEN0076 5 - 4
Coordonn ees sph eriques
Recherche du potentiel V dans lespace entre les deux conducteurs : V = V (r)
Probl ` eme :
R esoudre
2
V = 0 dans le domaine a r b
avec les conditions limites
_
V = V
0
pour r = a
V = 0 pour r = b
1
r
2

r
(r
2
V
r
) = 0 V =
A
r
+B
C.L V = V
0
1
r

1
b
1
a

1
b
Champ electrique

E

E =

V =
V
r
r =
V
0
1
a

1
b
1
r
2
r
ELEN0076 5 - 5
Courant total I
S
G
c
V
0
I
V
0

J =

E =
V
0
1
a

1
b
1
r
2
r = J(r) r
I =
_
S

J

dS =
V
0
1
a

1
b
4
S = surface dune sph` ere de rayon r
Conductance G
c
et r esistance :
G
c
=
I
V
0
=
4
1
a

1
b
R
c
=
1
G
c
=
1
4
(
1
a

1
b
)
ELEN0076 5 - 6
Champs variables - Pertes totales
Pour des champs variables : apparition de pertes li ees ` a la polarisation du
di electrique : pertes dues ` a la friction entre les dip oles induits par la polarisation
Ces pertes sont mod elis ees
` a l echelle microscopique par lajout dune partie imaginaire ` a la permittivit e :
=

pol
` a l echelle macroscopique par une conductance G
pol
Prise en compte ` a la fois des pertes par effet Joule et de polarisation :
permittivit e effective :
=

j(

pol
+

) =

tot
conductance totale equivalente :
G = G
c
+G
pol
ELEN0076 5 - 7
Admittance equivalente dun condensateur
Exemple : condensateur plan
G C Y = j
S
d
=
S
d
+

pol
S
d
+j

S
d
=

tot
S
d
+j

S
d
= G
c
+G
pol
+jC = G+jC
Pour une g eom etrie diff erente : f(d) facteur d ependant de la g eom etrie des conducteurs
(C plan : f(d) =
S
d
)
Y = j f(d) = f(d) +

pol
f(d) +j

f(d) =

tot
f(d) +j

f(d)
= G
c
+G
pol
+jC = G+jC
ELEN0076 5 - 8
5.2 Condensateur avec pertes
D eterminez la conductance dun condensateur plan au mica aux fr equences f
1
= 50 Hz
et f
2
= 1 GHz. Les caract eristiques du condensateur sont les suivantes
Surface des plateaux : S = 10 cm
2
S eparation des plateaux : d = 0.1 cm
Conductivit e du mica : = 10
15
S/m
Propri et es di electriques ` a f = 50 Hz :

= 6
0
et pertes di electriques n egligeables
Propri et es di electriques ` a f = 1 GHz :

= 6
0
et

tot
= 1.6 10
3

0
.
ELEN0076 5 - 9
1. ` a 50 Hz
G = G
c
=
S
d
= 10
15
S , G
pol
n egligeable
2. ` a 1 GHz
G = G
c
+G
pol
= (

pol
+

)
S
d
=

tot
S
d
= 89 10
6
S
On remarque quen hautes fr equences, G
c
repr esente une partie n egligeable de la
conductance totale.
ELEN0076 5 - 10
5.3 Blindage electrique
Un ecran m etallique est plac e entre les deux plateaux dun condensateur ` a air.
Le syst ` eme est aliment e par un g en erateur de
tension sinusodale (amplitude de tension V
0
,
fr equence f). Au moment du montage et avant
la mise en service du g en erateur, l ecran est
non-charg e. La surface des plateaux est egale
` a S.
D eterminez
a. les courants

I
1
et

I
2
,
b. lallure des lignes de champ electrique,
c. la distribution des charges sur l ecran,
dans chacun des deux cas suivants :
1. l ecran est ` a un potentiel ottant (il est isol e galvaniquement des autres el ements du
syst ` eme),
2. l ecran est reli e ` a la masse.
ELEN0076 5 - 11
1. Ecran ` a un potentiel ottant
Circuit electrique equivalent
+
R

I
2

I
1
C/2
C C

V
R
L ecran forme avec les deux plaques deux condensateurs de capacit e C =

0
S
d
connect es en s erie.

I
1
=

I
2
=
V
0
2R +
2
jC
ELEN0076 5 - 12
Lignes de champ
-

E = 0 ` a lint erieur de l ecran


-
-
-
-
-
-
-
+
+
+
+
+
+
+
+
La charge totale sur l ecran reste nulle.
Conclusion : l ecran nemp eche pas l electrisation du plateau droit du condensateur
par le plateau gauche.
ELEN0076 5 - 13
2. Ecran reli e ` a la masse
Circuit electrique equivalent
+
C C
R

I
2

I
1

V
R

I
2
= 0

I
1
=
V
0
R +
1
jC
ELEN0076 5 - 14
Lignes de champ
-
les charges circulent via la masse
-
-
-
-
-
-
-
Conclusion : l ecran remplit son r ole de blindage, l etat electrique du plateau droit ne
d epend plus de celui du plateau gauche.
ELEN0076 5 - 15
5.4 Pertes dun bobinage : variation en fonction de la fr equence
On consid` ere un bobinage ` a noyau ferromagn etique. Comment varient les pertes
suivantes en fonction de la fr equence ?
1. Les pertes par hyst er esis.
2. Les pertes dues aux courants de Foucault.
Dans chaque cas, on exprimera la puissance dissip ee P sous la forme P f
n
, o` u n est
la puissance alg ebrique correspondante.
ELEN0076 5 - 16
1. Pertes par hyst er esis
Energie dissip ee par cycle dans le mat eriau
_
cycle
H dB
Pour 1 s : f cycles
P
hyst
f
2. Pertes dues aux courants de Foucault
F.e.m cr e ee par la variation du ux : V =
d
dt
f
Pertes donn ees par RI
2
=
V
2
R
avec R la r esistance
du mat eriau
P
Foucault
f
2
ELEN0076 5 - 17
TP6 : ONDES
ELEN0076 6 - 1
Ondes planes, milieu lin eaire, homog` ene et isotrope non dissipatif
Equations dondes : milieu d epourvu de sources ( et

J)

E
t
2
= 0
2

H

2

H
t
2
= 0
Solution g en erale : onde plane progressive, polarisation transverse, r egime
harmonique

E(z, t) = E
0
cos(t kz) x = E
0
cos(k(z vt)) x

H(z, t) = E
0
k

cos(t kz) y
_


E =


H
t
_
Caract eristiques de la propagation
nombre donde : k =

, vitesse de propagation : v =

k
=
1

imp edance caract eristique du milieu de propagation


=
E
x
H
y
=

k
=
_

ELEN0076 6 - 2
Dans le vide :
v = c =
1

0
= 3 10
8
m/s =
0
=
_

0
= 377
Densit e de puissance transport ee : vecteur de Poynting :

S(z, t) =

E

H =
E
2
0

cos
2
(t kz) z (W/m
2
)
Transport de puissance selon z
x

E
z
y
ELEN0076 6 - 3
Dans le domaine fr equentiel : emploi des phaseurs

E(z, t) = ReE
0
e
j(tkz)
x = Re

E(z)e
jt
avec

E(z) = E
0
e
jkz
x
equations dondes :

E +
2

E = 0
2

H +
2

H = 0
champs

E(z) = E
0
e
jkz
x

H(z) =
E
0

e
jkz
y
densit e de puissance moyenne transport ee

S =
1
T
_
T

S(z, t)dt =
1
2
Re

=
E
2
0
2
z
ELEN0076 6 - 4
5.5 Puissance solaire rec ue par Mercure
La puissance du rayonnement solaire frappant la surface de Mercure vaut
approximativement S = 0.87 W/cm
2
.
1. En supposant que les ondes electromagn etiques sont emises par le soleil de facon
isotrope, estimez la puissance totale rayonn ee par le soleil.
2. En assimilant le rayonnement ` a la surface de mercure ` a une onde plane, estimez la
valeur efcace du champ electrique produit par ce rayonnement.
Donn ee : distance Mercure - Soleil, d = 60 Gm.
ELEN0076 6 - 5
Densit e de puissance du rayonnement emis ` a une distance d = 60 10
9
m =
amplitude du vecteur de Poynting [

S[
Puissance totale emise :
P =
_
S

S

dS = 4d
2
[

S[ = 3.94 10
26
W
S = surface dune sph` ere de rayon d centr ee sur le soleil
La puissance totale emise est la m eme quelle que soit la distance d consid er ee :
milieu non dissipatif
Ondes planes ` a la surface de Mercure : intensit e du vecteur de Poynting :
[

S[ =
E
2
0
2
0
= 0.87 10
4
E
rms
=
E
0

2
= 1.8 10
3
V/m
ELEN0076 6 - 6
Ondes planes dans un milieu dissipatif
Permittivit e effective
=

tot
Nombre donde : jk = j

= +j
Imp edance intrins` eque du milieu:
=
_


= r +jx = [[e
j

ELEN0076 6 - 7
Champs :

E = E
0
e
jkz
x = E
0
e
z
e
jz
x

H =
E
0

e
jkz
y =
E
0
[[
e
z
e
jz
e
j

E(z, t) = E
0
e
z
cos(t z) y

H(z, t) =
E
0
[[
e
z
cos(t z

) y
Coefcient datt enuation , vitesse de phase v = /
ELEN0076 6 - 8
Milieux bons conducteurs : effet de peau
Hypoth` eses :

pol
0 ,
Nombre donde :
jk (1 +j)
_

2
= (1 +j)
1

=
_
2

: epaisseur de peau
Coefcient datt enuation = 1/

f : dispersion !
Vitesse de phase : v =

f
Imp edance :
= Z
s

1 +j

= R
s
+jL
s
ELEN0076 6 - 9
5.6 Absorption dune onde plane par une plaque de cuivre
Une onde plane de fr equence f = 1 GHz se propage dans lair et frappe une plaque de
cuivre, ` a incidence normale.
z
S
i
H
i
E
i
cuivre,
0
,
0
,
vide,
0
,
0
, = 0
S
r
E
r
H
r
E
t
H
t
S
t
x
y
Sachant que la valeur de cr ete du champ electrique de londe incidente vaut
E
0
= 1 V/m et que la conductivit e electrique du cuivre vaut = 58 10
6
S/m,
d eterminez la valeur moyenne de la puissance absorb ee par unit e de surface par le
cuivre.
ELEN0076 6 - 10
R eexion et transmission dune onde plane ` a incidence normale sur un plan
conducteur
3 ondes planes ` a incidence normale sur la paroi du conducteur :
Onde incidente : se propage selon +z

E
i
= E
0
e
jk
i
z
x

H
i
=
E
0

0
e
jk
i
z
y
Nombre donde : jk
i
= j

0
Imp edance : =
0
=
_

0
= 377
Onde r e echie : se propage selon z

E
r
= E
0
e
jk
r
z
x

H
r
=
E
0

0
e
jk
r
z
y
Nombre donde : jk
r
= j

0
Imp edance : =
0
=
_

0
= 377
= coefcient de r eexion
=

E
r

E
i

z=0
ELEN0076 6 - 11
Onde transmise dans le conducteur : se propage selon +z

E
t
= E
0
e
jk
t
z
x

H
t
=
E
0

e
jk
t
z
y
Milieu dissipatif :
Nombre donde : jk
t
= j

0
Imp edance : =
_

0

= coefcient de transmission
=

E
t

E
i

z=0
Milieu bon conducteur ?

=
58 10
6
28.85 10
3

Epaisseur de peau
=
_
2

= 2.09 10
6
m
jk
t

1 +j

= Z
s

1 +j

= 8.25 10
3
(1 +j)
ELEN0076 6 - 12
Conditions aux limites sur la paroi conductrice : pour z = 0
continuit e de la composante tangentielle de

E :
(

E
i
+

E
r
)[
z=0
=

E
t
[
z=0
E
0
+E
0
= E
0
continuit e de la composante tangentielle de

H :
(

H
i


H
r
)[
z=0
=

H
t
[
z=0
E
0

E
0

0
=
E
0
Z
s
On d eduit :
=
Z
s

0
Z
s
+
0
= 1 +j4.38 10
5
=
2Z
s
Z
s
+
0
= 4.38 10
5
(1 +j)
On remarque :
[Z
s
[
0
1 ,
_
E
i
E
r
H
r
H
i
E
t
0 H
t
2H
i
ELEN0076 6 - 13
Puissance moyenne transport ee par londe incidente :
S
i
=
1
2
Re(

E
i

H
i

) =
1
2
E
2
0

0
= 132 mW/m
2
Puissance moyenne transport ee par londe r e echie :
S
r
=
1
2
Re(

E
r

H
r

) =
1
2
[[
2
E
2
0

0
Ces deux puissances moyennes sont ind ependantes de z (milieu non dissipatif)
Puissance moyenne absorb ee par le cuivre = puissance transmise (en z = 0):
S
t
=
1
2
Re(

E
t

H
t

) =
1
2
[[
2
E
2
0
[Z
s
[
2
Re(Z
s
) = 116 10
9
W/m
2
En utilisant lapproximation H
t
2H
i
:
S
t
=
1
2
Re(Z
s
)
_
2E
0

0
_
2
= 116 10
9
W/m
2
ELEN0076 6 - 14
5.7 Application du th eor` eme de Poynting ` a un bloc conducteur
On consid` ere un bloc de cuivre semi-inni s etendant dans lespace z > 0, et une onde
plane incidente venant frapper le bloc ` a incidence normale.
Etablissez lexpression de la puissance dissip ee dans le bloc de cuivre ` a partir du vecteur
de Poynting de londe transmise dans le bloc. Exprimez ensuite cette puissance en
fonction de lamplitude du champ magn etique ` a la surface du bloc et v eriez que le
r esultat obtenu est identique ` a celui etabli au cours par int egration directe de la puissance
dissip ee par effet Joule.
ELEN0076 6 - 15
1. Calcul par int egration directe de la puissance dissip ee par effet Joule : voir cours
Champ electrique et densit e de courant dans le cuivre :

E = E
0
e
(1+j)z/
x

J = E
0
e
(1+j)z/
x
Puissance dissip ee par effet Joule dans le bloc :
P =
1
2
_
V
Re(

)d
3
r
=
1
2
R
s
x
y
[

I[
2
avec
- R
s
=
1

: la r esistance par carr e


-

I : le courant total traversant le bloc
Tout se passe comme si le courant se r epartissait de mani ` ere uniforme sur une
epaisseur du bloc
ELEN0076 6 - 16
2. Calcul ` a partir du vecteur de Poynting ` a la surface du bloc
Puissance totale dissip ee dans le bloc = puissance p en etrant dans le bloc en z = 0
P =
1
2
_
x
_
y
Re(

E
t

t
) zdxdy
=
1
2
Re
_
x
_
y
(Z
s

H
0

H

0
)dxdy
=
xy
2
R
s
[

H
0
[
2
avec [

H
0
[ lamplitude du champ magn etique ` a la surface du bloc conducteur.
Il faut :
xy
2
R
s
[

H
0
[
2
=
1
2
R
s
x
y
[

I[
2
[

H
0
[ =
[

I[
y
ELEN0076 6 - 17
3. V erication par application du th eor ` eme dAmp` ere
D

H
0

J
z
y
x
C
B
A
Application du th eor ` eme dAmp` ere sur
le contour ABCD (C,D )
_
ABCD

H

d =
_
S

J

dS
= y
_

0

J dz
=

I =

H
0
y
_
B
A

H

d =

H
0
y
_
C
B

H

d =
_
A
D

H

d = 0 (

H

d)
_
D
C

H

d = 0 (

H 0 pour z )
ELEN0076 6 - 18
TP7 : LIGNES DE TRANSMISSION
PARAMETRES UNITAIRES
CARACTERISTIQUES DE LA PROPAGATION
ELEN0076 7 - 1
Equations des t el egraphistes en r egime harmonique
Hypoth` ese : OTEM, champs

E ,

H direction de propagation z
Mod elisation de la ligne
Z dz
I(z +dz)

V (z)

I(z)
Y dz
dz
V

H

E
I
I
z

V (z +dz)
Equations des t el egraphistes
Lois de Kirchhoff sur le troncon permettent dobtenir

V (z)
z
= Z()

I(z)

2

V (z)
z
2
= Z()Y ()

V (z)

I(z)
z
= Y ()

V (z)

2

I(z)
z
2
= Z()Y ()

I(z)
ELEN0076 7 - 2
Param` etres unitaires ou lin eiques
Y = G+jC :
C (pF/m): capacit e associ ee ` a la distribution du champ electrique dans le
di electrique s eparant les deux conducteurs
G (S/m) : pertes ohmiques (courant de conduction selon x) et de polarisation dans
le di electrique
Z() = R +jL :
L (H/m): inductance totale li ee ` a la distribution du champ magn etique ` a lint erieur
(des) et entre les conducteurs; somme des inductance internes (inu-
ence de leffet de peau) et externe
R (/m) : pertes ohmiques (courants selon z) dans le conducteur (inuence de
leffet de peau)
ELEN0076 7 - 3
Ondes V et I et param` etres de la propagation
Solutions g en erales des equations des t el egraphistes :

V (z) =

V
+
e
z
+

V

e
z

I(z) =

V
+
Z
0
e
z

Z
0
e
z
Somme d une onde incidente ou progressive se propageant selon +z et dune onde
r e echie se propageant selon z
Expression temporelle
v(z, t) = [

V
+
[e
z
cos((t
z
v
) +


V
+
)) +[

[e
z
cos((t +
z
v
) +


V

)
Imp edance caract eristique :
Z
0
= R
0
+jX
0
=
_
Z
Y
=

R +jL
G+jC
=

V
inc
(z)

I
inc
(z)
ELEN0076 7 - 4
Constante de propagation
= +j =

ZY =
_
(R +jL)(G+jC)
(Np/m) : coefcient datt enuation (rad/m) : constante de phase
Vitesse de phase et longueur donde :
v =

=
2

=
v
f
Cas particuliers :
Z
0

Ligne id eale R = G = 0
_
L
C
0

LC
Ligne ` a faibles pertes G C, R L
_
L
C
1
2
_
GZ
0
+
R
Z
0
_

LC
ELEN0076 7 - 5
6.1 Param` etres unitaires dune ligne t el ephonique
Une ligne t el ephonique de type bilaire est constitu ee de deux conducteurs cylindriques
de rayon a = 0.2 mm plac es parall ` element lun ` a lautre ` a une distance d = 0.6 mm.
Les ls sont isol es electriquement par un plastique (
r
= 2). Les ls sont en cuivre
( = 5.8 10
7
S/m,
r
= 1 et
r
= 1).
A la fr equence de travail f = 1 kHz, on a mesur e les quatre param` etres unitaires
suivants : R = 100 /km, C = 0.051 F/km, L = 0.6 mH/km et G 0 S/m.
1. Cette ligne travaille-t-elle en r egime de haute ou de basse fr equence ?
2. Travaille-t-elle en r egime de faibles pertes ?
3. D eterminez limp edance caract eristique, latt enuation lin eique et la vitesse de phase.
ELEN0076 7 - 6
1. Haute ou basse fr equence ?
Basse fr equence : courant uniform ement r eparti dans les conducteurs
Haute fr equence : effet pelliculaire, epaisseur de peau dimension du conducteur
Epaisseur de peau ` a la fr equence de f = 1 kHz :
=
_
2

= 2 mm a BF
2. Faibles pertes ?
R
L
=
100
210
3
0.6 10
3
= 26.5 1 non faibles pertes
ELEN0076 7 - 7
3. Imp edance caract eristique :
Z
0
=

R+jL
jC

R
jC
=
1 j

2
_
R
C
= 395(1 j)
Constante de propagation :
=
_
jC(R +jL)
_
jRC =
1 +j

RC = 0.12(1 +j) km
1
Att enuation:
= Re() = 0.12 Np/km
Vitesse de phase :
v =

= 52 10
3
km/s
ELEN0076 7 - 8
6.2 Param` etres unitaires dune ligne microbande
Une ligne microbande (microstrip en anglais) est constitu ee dun substrat isolant et
di electrique, m etallis e sur ses deux faces avec de laluminium. La face sup erieure
v ehicule un courant allant du g en erateur ` a la charge, le retour seffectuant par la face
inf erieure.
D eterminez les quatre param` etres unitaires C,
L, R et G, limp edance caract eristique Z
0
et
le coefcient datt enuation de cette ligne.
Donn ees :
Fr equence de travail : f = 18 GHz.
Aluminium : = 3.72 10
7
S/m,
r
= 1 et
r
= 1.
Di electrique : Si0
2
, et pertes di electriques n egligeables,
r
= 3.8 et
r
= 1.
ELEN0076 7 - 9
OTEM : dans un plan z = Cte : champ

E r eparti comme dans les probl ` emes
d electrostatique, champ

H r eparti comme dans les probl ` emes de magn etostatique (dans
lespace s eparant les deux conducteurs).
1. Calcul de C
+

E
- - - - - - - - - - - - - - - - -
+ + + + + + + + + + + +
Effets de bord n eglig es, C dun condensateur plan
C =
w
d
= 168 pF/m
2. Calcul de G
et pertes di electriques n egligeables G = 0
ELEN0076 7 - 10
3. Calcul de L
ext
Inductance externe : due au champ

H existant entre les conducteurs
z
y
x
c

H = 0

H
I
I
S
Effets de bord n eglig es :

H = H(x) y
Application du th eor ` eme dAmp` ere sur le contour c
_

H

d = I = wH

H =
I
w
y
Flux dinduction magn etique ` a travers la surface S de
1m de longueur:
=
_
S

B

dS =
Id
w
L
ext
=

I
=
d
w
= 0.25H/m
ELEN0076 7 - 11
4. Imp edance des conducteurs ` a la fr equence f = 18 GHz
HF ou BF ? Epaisseur de peau
=
_
2

= 0.62 m h HF
Bloc conducteur, imp edance par carr e : Z
s
=
1+j

Pour un conducteur de largeur w et de 1m de long (x = 1 , y = w):


Puissance dissip ee par effet Joule :
1
2
1
w
[

I[
2
=
1
2
R
1
[

I[
2
R esistance de 1m de conducteur : R
1
=
1
w
= 4.35 k/m
Energie emmagasin ee sous forme magn etique :
W
M
=
1
2
1
w
[

I[
2
=
1
2
L
1
[

I[
2
Inductance interne de 1 m de conducteur : L
1
=
1
w
= 39 nH/m
ELEN0076 7 - 12
5. Param` etres unitaires
C =
w
d
= 168 pF/m
L = L
ext
+ 2L
1
=
d
w
+
2
w
= 0.33 H/m
G = 0
R = 2R
1
=
2
w
= 8.7 k/m
ELEN0076 7 - 13
6. Param` etres carat eristiques de la propagation :
Faibles pertes ? oui
R
L
= 0.233 1
G
C
= 0
Imp edance caract eristique :
Z
0

_
L
C
= 44.3
Att enuation :

R
2Z
0
= 98.2 Np/m
ELEN0076 7 - 14
6.3 Param` etres unitaires dun c able coaxial
Un c able coaxial pr esente les param` etres suivants :
Param` etres g eom etriques : a = 1 mm, b = 1 cm et d = 1 mm.
Conducteurs : cuivre, = 5.8 10
7
S/m, =
0
et =
0
.
Di electrique : polystyr ` ene, = 10
16
S/m, =
0
, et

= 2.5
0
,

pol
/

= 10
5
` a f = 1 kHz ;

pol
/

= 2 10
4
` a f = 1 GHz
On demande de d eterminer les param` etres unitaires C, L, R et G, limp edance
caract eristique Z
0
et le coefcient datt enuation de ce c able, aux deux fr equences f = 1
kHz et f = 1 GHz.
Interpr etez les diff erences observ ees.
ELEN0076 7 - 15
1. Calcul de C : param` etre ind ependant de la fr equence
Effets de bord n eglig es, C dun c able coaxial (voir exercice 2.5)
C =
2
ln
b
a
= 60.4 pF/m
3. Calcul de L
ext
Inductance externe : due au champ

H existant entre les conducteurs, param` etre
ind ependant de la fr equence
L
ext
dun c able coaxial (voir exercice 4.6)
L
ext
=

2
ln
b
a
= 0.46 H/m
ELEN0076 7 - 16
3. Calcul de G: li e au courant de conduction dans le di electrique et aux pertes de
polarisation
permittivit e complexe :
=

tot
=

j(

pol
+

)
conductance (voir exercice 5.1)
G =

tot
f(d) =

tot
2
ln
b
a
= ( +

pol
)
2
ln
b
a

diff erents ` a f = 1 kHz et 1 GHz


f = 1 kHz f = 1 GHz

pol
= 1.4 10
12

pol
= 1.1 10
5

pertes di electriques dominent


G = 3.8 10
12
S/m G = 75.7 S/m
ELEN0076 7 - 17
4. Imp edance des conducteurs
HF ou BF ? Epaisseur de peau =
_
2

f = 1 kHz , = 2 mm > a, d : BF
f = 1 GHz , = 2 m a, d : HF
Bloc conducteur cylindrique, imp edance pour un m de conducteur : Z = R+jL
BF - 1 kHz HF - 1 GHz
courant uniform ement r eparti sur la
section
courant uniform ement r eparti sur
epaisseur de peau
R
1
=
1
a
2
R
1
=
1
2a
R
2
=
1
((d+b)
2
b
2
)
R
2
=
1
2b
L
1
=

c
8
L
1
=
R
1

L
2
= L
cext
L
2
=
R
2

voir exercice 4.6


ELEN0076 7 - 18
5. Param` etres unitaires
f = 1 kHz
C =
2
ln
b
a
= 60.4 pF/m
L = L
ext
+L
1
+L
2
=

2
ln
d
a
+

c
8
+

c
2
_
c
4
(c
2
b
2
)
2
ln
b
c

1
4
b
2

3
4
c
2
c
2
b
2
_
L = 0.52 H/m avec c = b +d
G = 3.8 10
12
S/m
R = R
1
+R
2
=
1
a
2
+
1
((b +d)
2
b
2
)
= 5.8 m/m
ELEN0076 7 - 19
f = 1 GHz
C =
2
ln
b
a
= 60.4 pF/m
L = L
ext
+L
1
+L
2
=

2
ln
d
a
+
1
2
(
1
a
+
1
b
) = 0.46 H/m L
ext
G = 75.7 S/m
R = R
1
+R
2
=
1
2
(
1
a
+
1
b
) = 1.5 /m
ELEN0076 7 - 20
6. Param` etres carat eristiques de la propagation :
` a f = 1 kHz
Faibles pertes ? non
R
L
1
Imp edance caract eristique :
Z
0
=

R+jL
G+jC
= 113.9 j66.4
Att enuation :
=
_
(R+jL)(G+jC) = 33 10
6
+j33 10
6
= Re() = 33 10
6
Np/m
ELEN0076 7 - 21
` a f = 1 GHz
Faibles pertes ? oui
R
L
= 5.2 10
4
1
G
C
= 2 10
4
1
Imp edance caract eristique :
Z
0

_
L
C
= 87.3
Att enuation :

R
2Z
0
+
Z
0
G
2
= 0.012 Np/m
ELEN0076 7 - 22
TP8 : LIGNES DE TRANSMISSION EN
REGIME HARMONIQUE
ONDES STATIONNAIRES - ADAPTATION
ELEN0076 8 - 1
Ligne termin ee sur une imp edance de charge Z
L
z
0

Z
0
Z
L
Ondes de tension et de courant:

V (z) =

V
+
e
z
+

e
z
=

V
+
(e
z
+e
z
)

I(z) =

V
+
Z
0
e
z

Z
0
e
z
Coefcient de r eexion ( ` a la charge):
=

V
+
=
Z
L
Z
0
Z
L
+Z
0
Imp edance vue de lentr ee de la ligne :
Z
in
() =

V (z = )

I(z = )
= Z
0
Z
L
cosh() +Z
0
sinh()
Z
0
cosh() +Z
L
sinh()
Cas de la ligne id eale :
Z
in
() = Z
0
Z
L
+jZ
0
tan()
Z
0
+jZ
L
tan()
ELEN0076 8 - 2
Cas particuliers
Z
L
Z
in

Ligne adapt ee Z
0
Z
0
0
Ligne cc. Z
L
= 0 jZ
0
tan() -1
Ligne ouverte Z
L
= jZ
0
cot() 1
ELEN0076 8 - 3
Lignes id eales : ondes stationnaires
Coefcient de r eexion g en eralis e :
(z) =

V
refl
(z)

V
inc
(z)
=

e
jz

V
+
e
jz
= e
2jz
= [[e
j(2z+

)
ELEN0076 8 - 4
Allure de [

V (z)[ le long de la ligne

V (z) =

V
+
e
jz
(1 + (z))
[

V (z)[ = [

V
+
[[1 + (z)[ = [

V
+
[
_
1 +[[
2
+ 2[[ cos(2z +

)
z
min
z
max
V
max
= [

V
+
[(1 +[[) V
min
= [

V
+
[(1 [[)
ELEN0076 8 - 5
Phase des ondes incidente et r e echie :
onde r e echie

z
min
z
max z
min
z
max
onde incidente
ELEN0076 8 - 6
Maxima=ventre de la tension lorsque onde incidente et onde r e echie sont en phase
z = z +

+ 2k z
max
=

4

2
Deux maxima successifs sont s epar es se /2
Minima=noeud de la tension lorsque onde incidente et onde r e echie sont en
opposition de phase
z = z +

+ (2k + 1) z
max
=

4

2


4
Deux minima successifs sont s epar es se /2
Un maximum et un minimum sont s epar es de /4
Taux dondes stationnaires :
S =
V
max
V
min
=
1 +[[
1 [[
[[ =
S 1
S + 1
ELEN0076 8 - 7
Courant et imp edance

I(z) =

V
+
Z
0
(e
jz
e
jz
) =

V
+
Z
0
e
jz
(1 (z))
En un maximum de tension, ondes incidente et r e echie de courant sont en
opposition de phase minimum de courant (Z
0
r eel)
I
min
=
[V
+
[
Z
0
(1 [[)
Z
max
=

V (z
max
)

I(z
max
)
= Z
0

V
+
(1 +[[)

V
+
(1 [[)
= SZ
0
imp edance r eelle !
En un mimimum de tension, ondes incidente et r e echie de courant sont en phase
maximum de courant
I
max
=
[V
+
[
Z
0
(1 +[[)
Z
min
=

V (z
min
)

I(z
min
)
= Z
0

V
+
(1 [[)

V
+
(1 +[[)
=
Z
0
S
imp edance r eelle !
ELEN0076 8 - 8
6.4 Allure de lamplitude de tension le long dune ligne en r egime harmonique
Une ligne de transmission id eale dimp edance caract eristique Z
0
= 75 et de longueur
= 0.6 est termin ee sur une charge dimp edance Z
L
= 100 +j150 .
1. D eterminez le taux dondes stationnaires S et la position des maxima et minima de la
tension le long de la ligne.
2. Esquissez lallure de [

V (z)[.
3. D eterminez limp edance vue au droit de lemplacement z = 0.4, limp edance
dentr ee de la ligne ainsi que les imp edances Z
min
et Z
max
vues au droit des
minima et maxima de tension.
ELEN0076 8 - 9
1. Taux dondes stationnaires S et emplacement des maxima et minima de [V [
Coefcient de r eexion
=
Z
L
Z
0
Z
L
+Z
0
= 0.51 +j0.42 = 0.66e
j0.7
[[ S :
S =
V
max
V
min
=
1 +[[
1 [[
= 4.9

z
max
, z
min
:
z
max
=

1
2
+k

or =
2

z
max
=

4
+k

2
= 0.056 +k

2
Longueur de la ligne : = 0.6
2 maxima : z
max,1
= 0.056 (k = 0), z
max,2
= 0.556 (k = 1)
1 minimum : z
min,1
= z
max,1
+

4
= 0.306
ELEN0076 8 - 10
2. Allure de [

V (z)[
[

V (z)[ = [

V
+
[[1 + (z)[ = [

V
+
[
_
1 +[[
2
+ 2[[ cos(2z +

)
z

0.056
0.306 0.556
V
min
V
max
ELEN0076 8 - 11
3. Imp edances
Z
in
(z = ) = Z
0
Z
L
+jZ
0
tan()
Z
0
+jZ
L
tan()
Imp edance vue de z =
1
= 0.4

1
= 0.8 Z
in
= 22 +j47.6
Imp edance dentr ee de la ligne : = 0.6
= 1.2 Z
in
= 133.6 j165.7
Imp edance aux maxima de la tension :
Z
max
= SZ
0
= 367.5
Imp edance aux minima de la tension :
Z
min
=
Z
0
S
= 15.3
ELEN0076 8 - 12
6.5 Mesure dune imp edance en hautes fr equences
Une ligne id eale dimp edance caract eristique Z
0
= 50 alimente une imp edance de
charge Z
L
, que lon cherche ` a d eterminer. La mesure de la tension le long de la ligne
fournit un taux dondes stationnaires S = 3. Le premier minimum de tension est
egalement localis e ` a une distance z
min
= 0.33 de la charge.
D eterminez la valeur de limp edance de charge Z
L
.
ELEN0076 8 - 13
Principe
S [[

min

Z
L
S =
1 +[[
1 [[
[[ =
S 1
S + 1
=
1
2

min
=

4
+

4

= 0.32 = 1
= 0.5e
j1
Z
L
= Z
0
1 +
1
= 52.5 +j59.1
ELEN0076 8 - 14
6.6 Ligne id eale quart donde
On consid` ere un troncon de ligne id eale dimp edance caract eristique Z
0
et dune
longueur egale au quart de la longueur donde du signal transmis. Pour chacune des
situations suivantes :
1. ligne laiss ee ouverte ` a son extr emit e;
2. ligne court-circuit ee;
3. ligne termin ee sur une r esistance de charge R
1
< Z
0
;
4. ligne termin ee sur une r esistance de charge R
2
> Z
0
;
d eterminez :
le coefcient de reexion et le taux dondes stationnaires ;
lexpression et lallure du module de la tension et du courant le long de la ligne;
limp edance vue de lentr ee de la ligne.
ELEN0076 8 - 15
Ligne dune longueur dun quart de la longueur donde :
=

4
, =
2

4
=

2
1. Ligne ouverte.
Z
L
= , =
Z
L
Z
0
Z
L
+Z
0
= 1 S =
1 +[[
1 [[
=

V (z) =

V
+
e
jz
+

V
+
e
jz
= 2

V
+
cos z

I(z) =

V
+
Z
0
e
jz

V
+
Z
0
e
jz
=
2j

V
+
Z
0
sin z
Z
in1
=

V ()

I()
= 0
[

I(z)[
z
[

V (z)[
z
ELEN0076 8 - 16
2. Ligne court-circuit ee.
Z
L
= 0 , =
Z
L
Z
0
Z
L
+Z
0
= 1 S =
1 +[[
1 [[
=

V (z) =

V
+
e
jz


V
+
e
jz
= 2j

V
+
sin z

I(z) =

V
+
Z
0
e
jz
+

V
+
Z
0
e
jz
=
2

V
+
Z
0
cos z
Z
in2
=

V ()

I()
=
[

I(z)[
z
[

V (z)[
z
ELEN0076 8 - 17
3. R esistance de charge R
1
< Z
0
.
Z
L
= R
1
, =
R
1
Z
0
R
1
+Z
0
< 0

=
[

V (z)[ = [

V
+
[
_
1 +[[
2
+ 2[[ cos(2z +)
maximum en z
max
=

4
minimum en z
min
= 0
[

I(z)[ =
[

V
+
[
Z
0
_
1 +[[
2
+ 2[[ cos(2z)
maximum en z
max
= 0 minimum en z
min
=

4
Z
in3
= Z
0
R
1
+jZ
0
tg

2
Z
0
+jR
1
tg

2
=
Z
2
0
R
1
> Z
0
[

I(z)[
z
[

V (z)[
z
ELEN0076 8 - 18
4. R esistance de charge R
2
> Z
0
.
Z
L
= R
2
, =
R
2
Z
0
R
2
+Z
0
> 0

= 0
[

V (z)[ = [

V
+
[
_
1 +[[
2
+ 2[[ cos(2z)
maximum en z
max
= 0 minimum en z
min
=

4
[

I(z)[ =
[

V
+
[
Z
0
_
1 +[[
2
+ 2[[ cos(2z +)
maximum en z
max
=

4
minimum en z
min
= 0
Z
in4
= Z
0
R
2
+jZ
0
tg

2
Z
0
+jR
2
tg

2
=
Z
2
0
R
2
< Z
0
[

I(z)[
z
[

V (z)[
z
On remarque que dans les situations 3 et 4 :
Z
L
.Z
in
= Z
2
0
ELEN0076 8 - 19
6.7 Adaptation par tronc on de ligne quart donde
Une ligne de longueur
1
= 10 m et dimp edance caract eristique Z
01
= 300 doit etre
connect ee ` a une deuxi ` eme ligne, dimp edance caract eristique Z
02
= 150 , de
longueur
2
= 20 m, et termin ee sur une charge Z
L
= 150 .
Les lignes sont constitu ees de c ables coaxiaux remplis avec un di electrique de
permittivit e relative
r
= 2.25 et sont consid er ees sans perte. La fr equence de travail est
f = 50 MHz.
1. Si les deux lignes sont connect ees directement, d eterminez le taux dondes
stationnaires dans le troncon de longueur
1
.
Z
01
= 300 Z
02
= 150

1
Z
L
ELEN0076 8 - 20
2. On souhaite ` a pr esent connecter les deux lignes par linterm ediaire dun adaptateur
quart donde, constitu e dun c able coaxial rempli avec le m eme di electrique.
Z
0
Z
01
= 300 Z
02
= 150

1

4

2
Z
L
adaptateur
(a) D eterminez la longueur et limp edance caract eristique Z
0
du troncon
interm ediaire permettant de r ealiser ladaptation ` a la premi ` ere ligne.
(b) D eterminez le coefcient de r eexion au droit de lentr ee de la deuxi ` eme ligne
(imp edance caract eristique Z
02
) et au droit de lentr ee de ladaptateur. En
d eduire le taux dondes stationnaires dans le troncon de longueur
1
et celui dans
ladaptateur.
ELEN0076 8 - 21
1. Lignes en cascade sans adaptateur.
Imp edance vue ` a lentr ee du troncon de ligne 2 :
la ligne est adapt ee (Z
L
= Z
02
) et Z
in
(
2
) = Z
02
Taux dondes stationnaires dans le troncon de ligne 1:
la ligne 1 est termin ee sur limp edance equivalente Z
eq
= Z
in
(
2
) = 150
coefcient de r eexion correspondant :
=
Z
eq
Z
01
Z
eq
+Z
01
= 1/3
taux dondes stationnaires :
S =
1 +[[
1 [[
= 2
ELEN0076 8 - 22
2. Adaptation par troncon quart donde.
(a) Adaptation au troncon de ligne 1 si ` a lentr ee de ladaptateur Z
in
= Z
01
Ladaptateur est termin e sur limp edance equivalente Z
eq
= Z
02
. Pour une longueur
dadaptateur de

4
:
Z
in
=
Z
2
0
Z
02
On d eduit limp edance caract eristique de ladaptateur :
Z
0
=
_
Z
01
Z
02
= 212.1
Longueur de ladaptateur :
v
p
=
1

0
=
3 10
8

r
= 2 10
8
m/s , =
v
p
f
= 4 m, =

4
= 1 m
ELEN0076 8 - 23
(b) Coefcients de r eexion et taux dondes stationnaires
dans ladaptateur termin e sur Z
eq
= Z
02
=
Z
02
Z
0
Z
02
+Z
0
= 0.17 , S =
1 +[[
1 [[
= 1.4
dans le troncon de ligne 1 : ligne adapt ee
= 0 , S = 1
ELEN0076 8 - 24
TP9 : LIGNES DE TRANSMISSION EN
REGIME HARMONIQUE
ELEN0076 9 - 1
6.8 Adaptation dun r eseau dantennes par ajout dun stub
Un r eseau dantennes est constitu e de la mise en parall ` ele dun ensemble de 10 antennes
identiques, chacune aliment ee par une ligne id eale. Chaque antenne est adapt ee ` a sa
ligne dalimentation de sorte que limp edance caract eristique de chaque ligne
dalimentation Z
0
est egale ` a limp edance de lantenne Z
L
= 50 . On d esire r ealiser
ladaptation de ce r eseau dantennes ` a sa ligne dalimentation principale dimp edance
caract eristique Z
0
= 50 . A cette n, on place en parall ` ele sur la ligne dalimentation
principale, ` a une distance
1
du r eseau dantennes, un stub en court-circuit de m eme
imp edance caract eristique Z
0
et de longueur
2
.
D eterminez les valeurs de
1
et
2
qui r ealisent ladaptation. Pour ces valeurs, que vaut le
taux dondes stationnaires (i) dans la ligne au droit du r eseau dantennes, (ii) dans le stub
au droit du court-circuit ?
Donn ees :
Fr equence de travail : f = 1 GHz.
Constantes caract eristiques des lignes :
r
= 2.3 et
r
= 1.
ELEN0076 9 - 2
ELEN0076 9 - 3
Principe
Soit Y
1
(
1
) = G
1
(
1
) +jB
1
(
1
) ladmittance vue en z =
1
au droit du troncon
de ligne termin e sur le r eseau dantennes
Soit Y
cc
(
2
) = jY
0
cot(
2
) = jB
2
(
2
) ladmittance du stub cc. de longueur
2
Les deux el ements sont connect es en parall ` ele, ladmittance equivalente vaut :
Y = G
1
(
1
) +jB
1
(
1
) +jB
2
(
2
)
Pour r ealiser ladaptation, il faut : Y = Y
0
avec Y
0
r eel
1. D eterminer
1
tel que G
1
(
1
) = Y
0
2.
1
x e, on a : Y
1
= Y
0
+jB
1
avec B
1
connu
3. D eterminer
2
tel que B
2
(
2
) = B
1
ELEN0076 9 - 4
Imp edance du r eseau de 10 antennes en parall ` ele : Z
L
= Z
0
/10 = 5
Longueur donde :
=
v
f
= 19.8 cm avec v =
1

= 1.98 10
8
m/s
Admittance Y
1
Y
1
(
1
) = Y
0
Z
0
+jZ
L
tan(
1
)
Z
L
+jZ
0
tan(
1
)
Il faut :
Re(Y
1
) = Y
0
250 + 250 tan
2
(
1
) = 25 + 2500 tan
2
(
1
)
tan(
1
) =
_
1
10
deux solutions possibles pour
1
:

(a)
1
= 0.049 = 0.97 cm
(b)
1
= 0.451 = 8.93 cm
ELEN0076 9 - 5
On trouve
B
1
(
(a)
1
) = 2.82Y
0
B
1
(
(b)
1
) = 2.82Y
0
Solution (a) :
1
= 0.049
Il faut : B
2
(
2
) = 2.82Y
0
Y
0
cot(
2
) = 2.82Y
0

2
= 0.446 = 8.83 cm
Solution (b) :
1
= 0.451
Il faut : B
2
(
2
) = 2.82Y
0
Y
0
cot(
2
) = 2.82Y
0

2
= 0.054 = 1.07 cm
ELEN0076 9 - 6
Taux dondes stationnaires :
Dans le troncon de ligne [
1
, 0]:
=
Z
L
Z
0
Z
L
+ Z
0
= 0.82 S =
1 +[[
1 [[
= 10.1
Dans le stub cc. :
= 1 S =
Dans le troncon de ligne [,
1
]:
= 0 S = 1
Pas de r eexion, adaptation !
ELEN0076 9 - 7
6.9 Bilan de puissance dans un quart donde en r egime harmonique
Un g en erateur de tension efcace V
g
= 15 V et dimp edance interne Z
g
= 75
alimente une charge dimp edance Z
L
= 60 par linterm ediaire dune ligne de
transmission id eale dimp edance caract eristique Z
0
= 75 et de longueur = /4.
1. Que vaut limp edance dentr ee Z
in
() ?
2. Que vaut la puissance d elivr ee ` a la ligne ?
3. Quelle est la tension d evelopp ee aux bornes de la charge, en z = 0 ?
4. Quelle est la puissance dissip ee par la charge ?
5. Esquissez lallure de [

V (z)[ pour < z < 0.


ELEN0076 9 - 8
1. Imp edance dentr ee :
=

4
Z
in
() =
Z
2
0
Z
L
= 93.75
2. Circuit electrique equivalent : remplacer la ligne ferm ee sur Z
L
par limp edance
dentr ee Z
in
+

z =
Z
in
Z
g

V ()

I()

V
g

V () =

V
g
Z
in
Z
g
+Z
in
= 8.33 V

I() =

V
g
Z
g
+Z
in
= 88.9 mA
P
in
= Re(

V ()

()) = 0.74 W
ELEN0076 9 - 9
3. Exprimer les conditions aux limites ` a lentr ee de la ligne (au g en erateur) et ` a la
charge :
au g en erateur z = :

V () =

V
+
(e
j
+e
j
) = 8.33
avec =
Z
L
Z
0
Z
L
+Z
0
= 0.11

V
+
= 7.5j
` a la charge z = 0 :

V (0) =

V
+
(1 +) = j6.67
v
0
(t) =

2 6.67 cos(t

2
) V
4. Puissance consomm ee par la charge :
z = 0

V
L
=

V (0)

I
L
=

I(0) =

V
L
Z
L
Z
L
P
L
= Re(

V
L

L
) = Re(
V
2
L
Z

L
) = 0.74 W = P
in
!!
Ligne id eale, pas de pertes dans la ligne.
ELEN0076 9 - 10
5. Allure de [

V (z)[
[

V (z)[ = [

V
+
[
_
1 +[[
2
+ 2[[ cos(2z +) = 7.5
_
1.012 0.22 cos(
4

z)
z
max
=

4
= z
min
= 0
z/
[V [
On v erie :
S =
1 +[[
1 [[
=
1.11
0.89
=
8.33
6.67
ELEN0076 9 - 11
6.10 Adaptation de ligne et adaptation conjugu ee
On consid` ere le montage suivant, pour lequel on souhaite comparer plusieurs types
dadaptation.
Etant donn e limp edance dentr ee Z
in
= R
in
+jX
in
et limp edance interne du
g en erateur, Z
g
= R
g
+jX
g
,
1. etablissez lexpression de la puissance d elivr ee ` a la charge ;
2. parmi les deux sch emas dadaptation suivants : Z
L
= Z
0
(adaptation de ligne) et
Z
g
= Z
in
(adaptation de g en erateur), quel est celui qui garantit la plus grande
puissance d elivr ee ` a Z
L
?
3. D eterminez la valeur de Z
in
pour laquelle la puissance d elivr ee ` a la charge est
maximale (adaptation conjugu ee).
ELEN0076 9 - 12
1. Circuit equivalent :
+

z =
Z
in
Z
g

V ()

I()

V
g

I() =

V
g
Z
g
+Z
in

V () =
Z
in
Z
g
+Z
in

V
g
amplitudes en valeur efcace
Ligne id eale : P
in
= P
Z
L
P
Z
L
= Re(

V ()

())
= V
2
g
R
in
(R
g
+R
in
)
2
+ (X
g
+X
in
)
2
ELEN0076 9 - 13
2. a : adaptation de ligne : Z
L
= Z
0
, R
in
= Z
0
, X
in
= 0
P
a
= V
2
g
Z
0
(R
g
+Z
0
)
2
+ (X
g
)
2
2. b : adaptation du g en erateur : Z
g
= Z
in
P
b
= V
2
g
R
g
4R
2
g
+ 4X
2
g
Comparaison :
P
b
P
a
=
R
g
4Z
0
(Z
0
+R
g
)
2
+X
2
g
R
2
g
+X
2
g
R
g
4Z
0
P
b
< P
a
R
g
> 4Z
0
P
b
> P
a
Pour une faible imp edance interne du g en erateur, ladaptation ` a la ligne est
pr ef erable
ELEN0076 9 - 14
3. Adaptation conjugu ee
P
max
lorsque Z
in
= Z

g
(voir cours de circuits electriques, adaptation des imp edances en r egime sinusodal
etabli)
P
max
= V
2
g
1
4R
g
et P
max
> P
a
, P
max
> P
b
Il existe des r eexions qui interf ` erent de mani ` ere constructive pour augmenter la
puissance transmise ` a la charge.
ELEN0076 9 - 15
TP10 : TRANSITOIRES DANS LES LIGNES
DE TRANSMISSION
ELEN0076 10 - 1
7.1 Propagation dune impulsion carr ee dans une ligne id eale
+

V
g
R
L
R
g
V
g
1 V
1 ns
t
z
0
ligne id eale
Z
0
Une ligne id eale dimp edance caract eristique Z
0
= 50 , de longueur = 2 m et de
vitesse de phase v
p
= 10
8
m/s est termin ee sur une r esistance de charge R
L
= 20 ,
et aliment ee par un g en erateur de r esistance interne R
g
= 30 . Ce g en erateur d elivre
un signal sous forme dimpulsions carr ees de dur ee = 1 ns et de tension V
0
= 1 V.
On demande
1. de tracer lallure de la tension et du courant en fonction du temps, au droit de z = 0
et z = /2. On se limitera aux deux premi ` eres impulsions recues ` a ces positions ;
2. de calculer la fraction de l energie fournie au g en erateur qui est dissip ee dans la
charge R
L
.
ELEN0076 10 - 2
Lignes en temporel
Transmission de signaux ,= signaux sinusodaux : impulsion, echelon, ...
Spectre de fr equences large
Superposition : traiter propagation de chaque composante de Fourier
Sous l hypoth` ese dune ligne id eale:
vitesse v
p
ind ependante de : toutes les composantes fr equentielles se
propagent ` a la m eme vitesse, pas de distorsion du signal;
att enuation nulle : = 0
imp edance caract eristique r eelle
le signal se propage sans modication de forme
consid erer la propagation du signal temporel de forme inchang ee ` a la vitesse v
p
appliquer les conditions aux limites aux extr emit es de la ligne (z = 0 et z = )
on ne peut plus parler dimp edances Z, dadmittances Y ou de phaseurs !!
ELEN0076 10 - 3
1. t = 0
Une impulsion est emise par le g en erateur = onde incidente, pas encore donde
r e echie.
Conditions aux limites ` a lentr ee de la ligne : limpulsion voit limp edance
caract eristique Z
0
(r eelle !).
La ligne peut etre remplac ee par son imp edance caract eristique Z
0
+

z =
I()
V ()
Z
0
V
g
R
g
V (, 0) = V
+
= V
g
Z
0
R
g
+Z
0
= 0.625 V
I(, 0) = I
+
=
V
+
Z
0
= 12.5 mA
2. Londe se propage le long de la ligne : temps n ecessaire pour atteindre lextr emit e
z = 0 : T =

v
p
= 20 ns
Hypoth` ese : T : impulsion quasi instantan ee (pas de superposition dondes
le long de la ligne, uniquement aux extr emit es)
ELEN0076 10 - 4
3. t = T :
Limpulsion subit une r eexion sur la r esistance de charge R
L
en z = 0.
Conditions aux limites :
V
L
= R
L
I
L
= V (0, T) = V
+
+V

I
L
= I(0, T) = I
+
+I

=
V
+
Z
0

Z
0
On d eduit :
V

=
R
L
Z
0
R
L
+Z
0
V
+
=
L
V
+
= 0.43 V
+
= 0.268 V
I

=
V

Z
0
=
L
I
+
= 5.4 mA
Londe V
+
est r e echie avec le coefcient de r eexion
L
4. T < t < 2T :
Londe V

se propage vers le g en erateur.


ELEN0076 10 - 5
5. t = 2T :
Londe V

subit une r eexion sur le g en erateur avec un coefcient

g
=
R
g
Z
0
R
g
+Z
0
= 0.25
V
+2
=
g
V

=
L

g
V
+
= 0.067 V I
+2
=
g
I

=
L

g
I
+
= 1.34 mA
6. 2T < t < 3T :
Londe V
+2
se propage vers la charge.
6. t = 3T :
Londe V
+2
subit une r eexion sur la charge avec le coefcient
L
V
2
=
L
V
+2
=
2
L

g
V
+
= 0.027 V I
2
=
L
I
+2
=
2
L

g
I
+
= 0.6 mA
6. 3T < t < 4T :
Londe V
2
se propage vers le g en erateur.
Le processus de r eexions successives se poursuit ind eniment.
ELEN0076 10 - 6
Sch ema temporel des r eexions :
z
20
10
z = /2
V

+V
2+
= 0.201 V
I

+I
2+
= 6.74 mA
I
2+
+I
2
= 1.94 mA
V
2+
+V
2
= 38 mV
I
+
+I

= 17.9 mA
V
+
+V

= 0.357 V
V
2
V
2+
V

V
+
t (ns)
t
On v erie : (t > )
V (, t) +R
g
I(, t) = 0 V (0, t) = R
L
I(0, t)
ELEN0076 10 - 7
Tension et courant ` a la charge (z = 0)
60
0.357
t (ns)
I(0)
17.9
1.94
V (0)
t (ns)
0.038
60 20 20
Tension et courant ` a la moiti e de la ligne (z = /2)
t (ns)
50
5.4
1.34
12.5
10
0.067
-0.268
0.625
50 10
I(/2) V (/2)
t (ns)
ELEN0076 10 - 8
Energie dissip ee
Energie fournie par le g en erateur (1` ere impulsion) : E = V
g
I
+

Energie transport ee par la 1` ere impulsion incidente dans la ligne :


E
+
= V
+
I
+
= E
Z
0
Z
0
+R
g
La diff erence E E
+
est consomm ee par la r esistance R
g
: E E
+
= R
g
I
2
+

A la premi ` ere r eexion ` a la charge :


la fraction E

= V

=
2
L
E
+
est r e echie dans la ligne vers le
g en erateur
la fraction E
+
(1
2
L
) = V (0)I(0) est dissip ee dans la charge R
L
A la premi ` ere r eexion au g en erateur :
la fraction E
+2
= V
+2
I
+2
=
2
L

2
g
E
+
est r e echie dans la ligne vers la
charge
la fraction E

(1
2
g
) est dissip ee dans la charge R
g
ELEN0076 10 - 9
Diagramme des r eexions
temps t E dissip ee dans R
g
E dissip ee dans R
L
0 E
R
g
Z
0
+R
g
0 < t < T E
+
T (1
2
L
)E
+
T < t < 2T
2
L
E
+
2T (1
2
g
)
2
L
E
+
2T < t < 3T
2
L

2
g
E
+
3T (1
2
L
)
2
L

2
g
E
+
3T < t < 4T
4
L

2
g
E
+
4T (1
2
g
)
4
L

2
g
E
+
4T < t < 5T
4
L

4
g
E
+
5T (1
2
L
)
4
L

4
g
E
+
...
ELEN0076 10 - 10
Energie totale dissip ee dans R
L
E
L
= (1
2
L
)(1 +
2
L

2
g
+
4
L

4
g
+. . .)E
+
=
1
2
L
1
2
L

2
G
Z
0
Z
0
+R
g
E
fraction de l energie fournie par le g en erateur et consomm ee dans R
L
:
E
L
E
=
1
2
L
1
2
L

2
G
Z
0
Z
0
+R
g
= 51.5 %
Remarque : si la ligne etait adapt ee (R
L
= Z
0
), il ny aurait aucune r eexion
(
L
= 0); l energie dissip ee dans R
L
est donn ee par E
+
:
E
L
E
=
Z
0
Z
0
+R
g
= 62.5 %
ELEN0076 10 - 11
7.2 Propagation dun echelon de tension dans une ligne id eale
Une ligne id eale a pour param` etres Z
0
= 50 , = 2 m et v
p
= 2 10
8
m/s.
Elle est termin ee sur une r esistance de charge R
L
= Z
0
/2, et aliment ee par un
g en erateur de r esistance interne R
g
= Z
0
/2. Ce g en erateur d elivre un signal echelon
damplitude 1 V.
+

0
z
V
g
t = 0
R
g
= Z
0
/2
R
L
= Z
0
/2
Z
0
ligne id eale
Esquissez lallure de la tension en z = en fonction du temps durant les 50 premi ` eres
millisecondes apr ` es l etablissement de l echelon de tension.
ELEN0076 10 - 12
M eme sch ema de r esolution qu ` a lexercice 6.9 (circuit r esistif)
Coefcients de r eexion ` a la charge et au g en erateur

L
=
g
= =
1
3
puisque R
L
= R
g
=
Z
0
2
1. Propagation de l echelon le long de la ligne ` a la vitesse v
p
Onde incidente :
V
+
=
Z
0
Z
0
+R
g
V
0
=
2
3
V
0
V
+
z
V (z)
z =
Temps n ecessaire pour atteindre la charge : T = /v
p
= 10 ns
ELEN0076 10 - 13
2. t = T : R eexion sur la r esistance R
L
, onde r e echie se propageant vers le
g en erateur
V

= V
+
=
1
3
V
+
=
2
9
V
0
Principe de superposition : onde incidente et r e echie sadditionnent : T < t < 2T
V
+
+V

z
V (z)
z =
V
+
3. t = 2T : R eexion sur R
g
, onde r e echie se propageant vers la charge
V
+2
= V

=
2
V
+
=
2
27
V
0
2T < t < 3T
z =
z
V (z)
ELEN0076 10 - 14
Diagramme des r eexions
V
+
+V

+V
2+
V
2+
V

V
+
t (ns)
t
z
10
V
2
V
+
+V

+V
2+
+V
2
0.494 V
V
+
(1 + +
2
+
3
)
V
+
(1 +) = 0.444 V
V
+
+V

V
+
(1 + +
2
) = 0.519 V
ELEN0076 10 - 15
V (0, t)
T
t
0.5V
0
V (, t)
0.5V
0
t
T
Les tensions convergent nalement vers :
V (0, t ) =
2
3
V
0
(1 + +
2
+
3
+
4
+. . .)
=
2
3
V
0
1
=
1
2
V
0
= V (, t )
= r esultat circuit localis e, r egime continu etabli ( )
ELEN0076 10 - 16
7.3 Charge dun condensateur en r egime transitoire
Un g en erateur alimente un condensateur de capacit e C par linterm ediaire dune ligne de
transmission id eale. En supposant que linterrupteur est ferm e en t = 0 et que le
condensateur est initialement non charg e, d eterminez lallure de la tension V (z, t) le
long de la ligne.

+
t = 0
V
0
R
g
= Z
0
C
z
0
ligne id eale
Z
0
ELEN0076 10 - 17
Charge = condensateur : pas de coefcient de r eexion
Ecrire les equations temporelles exprimant les conditions aux limites ` a lextr emit e de
la ligne
1. t < T : Propagation de l echelon le long de la ligne ` a la vitesse v
p
.
Onde incidente :
V
+
=
Z
0
Z
0
+R
g
V
0
=
1
2
V
0
z =
V
0
/2
z
V (z)
Temps n ecessaire pour atteindre la charge : T = /v
p
ELEN0076 10 - 18
2. t = T : r eexion au condensateur. On a
V
C
= V
+
+V

I
C
=
V
+
Z
0

Z
0
I
C
= C
dV
C
dt
On d eduit :
dV
C
dt
+
V
C
Z
0
C
=
V
0
Z
0
C
La solution est :
V
C
(t) =
_
0 pour 0 < t < T, C non charg e initialement
V
0
(1 e
(tT)/
) pour t > T, avec = Z
0
C, Cte de temps
Londe r e echie vaut :
V

= V
C
V
+
=
V
0
2
(1 2e
(tT)/
) t > T
ELEN0076 10 - 19
3. t > T : Londe r e echie V

se propage vers le g en erateur et sajoute ` a londe


incidente V
+
Arriv ee au g en erateur, il y a adaptation (R
g
= Z
0
) et plus aucune r eexion
R esum e
0 < t < T , V
C
(t) = 0
z =
V
0
/2
z
V (z)
T < t < 2T , V
C
(t) = V
0
(1 e
(tT)/
)
V
0
/2
V
+
z
z =
V
+
+V

(z)
z =
V (z)
z
ELEN0076 10 - 20
t > 2T , V
C
(t) = V
0
(1 e
(tT)/
)
z
t
V
0
V
+
+V

z
V (z)
z = z =
V (z)
ELEN0076 10 - 21
TP11 : GUIDES DONDES
ELEN0076 11 - 1
Rappel : equations de Maxwell en rotationnel, r egime harmonique

E = j

H


E
z
y



E
y
z
= j

H
x
(1)


E
x
z



E
z
x
= j

H
y
(2)


E
y
x



E
x
y
= j

H
z
(3)

H = j

E


H
z
y



H
y
z
= j

E
x
(4)


H
x
z



H
z
x
= j

E
y
(5)


H
y
x



H
x
y
= j

E
z
(6)
ELEN0076 11 - 2
Guide dondes constitu e de 2 plaques conductrices //
,
x
z
y
Propagation selon z, effets de bord n eglig es, on-
des progressives

E =

E(x)e
z

H =

H(x)e
z
Eqs. de propagation dans le milieu di electrique :
_

2
x
2
+k
2
x
_
_

H
_
= 0 avec
k
2
x
=
2
+
2

=
2
+k
2
Conditions aux limites sur les plaques : n

E = 0 , n

B = 0
pour x = 0, x = a

E
y
= 0,

E
z
= 0,

H
x
= 0
3 types de modes de propagation
modes TE :

E z, (

E
y
,

H
x
,

H
z
)
modes TM :

H z, (

E
x
,

E
z
,

H
y
)
mode TEM :

E,

H z, (

E
x
,

H
y
)
ELEN0076 11 - 3
Modes TE
Solution g en erale pour

E
y

E
y
= (E
0
sin(k
x
x) +Bcos(k
x
x))e
z
C.L. :

E
y
= 0 pour x = 0, x = a B = 0, k
x
=
m
a
, m = 1, 2, . . .
Expression des champs :

E = E
0
sin(
mx
a
)e
z
y

H =

E
j
_

H
x
=

j
E
0
sin(
mx
a
)e
z

H
z
=
m/a
j
E
0
cos(
mx
a
)e
z
Constante de propagation

2
=
_
m
a
_
2

2
= (
2
c

2
)
pout <
c
: = : att enuation
pour >
c
: = j : propagation
ELEN0076 11 - 4
Caract eristiques de la propagation : mode TE
m
, >
c
Fr equence de coupure
f
c
= m
v
2a

c
=
2a
m
avec v =
1

v est la vitssse de propagation dans le milieu di electrique ind eni


Constante de propagation
= j = j

1
_
f
c
f
_
2
= j

1
_

c
_
2
vitesses de phase et de groupe
v
p
=

=
v
_
1
_
f
f
c
_
2
v
g
=
d
d
= v

1
_
f
f
c
_
2
ELEN0076 11 - 5
Modes TM
Solution g en erale pour

H
y

H
y
= (Asin(k
x
x) +H
0
cos(k
x
x))e
z
On d eduit

E
z
, composante tangentielle de

E (eq. (6)):

E
z
=
k
x
j
(Acos(k
x
x) H
0
sin(k
x
x))e
z
C.L. :

E
z
= 0 pour x = 0, x = a A = 0, k
x
=
m
a
, m = 0, 1, . . .
Expression des champs :

H = H
0
cos(
mx
a
)e
z
y

E =

H
j
_

E
x
=

j
H
0
cos(
mx
a
)e
z

E
z
=
m/a
j
H
0
sin(
mx
a
)e
z
Conditions et caract eristiques de la propagation identiques ` a celles des modes TE.
ELEN0076 11 - 6
Mode TEM
Cas particulier des modes TM = mode TM
0
Pas de fr equence de coupure :
k
x
= 0
2
+
2
= 0 = j = j

Champs :

H(z) = H
0
e
jz
y

E(z) =
_

H
0
e
jz
x
Ligne de transmission !!
ELEN0076 11 - 7
8.1 Propagation dun mode TEM dans un guide dondes coaxial
Un c able coaxial a les param` etres suivants :
rayon int erieur : a = 1.2 mm
rayon ext erieur : b = 3.25 mm
epaisseur du conducteur ext erieur : d = 2 mm
propri et es di electriques ` a f = 100 MHz :

r
= 1.3 et

r
0
conducteurs consid er es parfaits.
Le c able est aliment e par un signal sinusodal damplitude de cr ete V
0
= 500 V et de
fr equence f = 100 MHz. Ce signal se propage sous la forme dun mode TEM (onde
progressive).
1. Esquisser lallure des lignes de champ de

E et

H dans une section transverse.
2. D eterminer les expressions des phaseurs

E et

H en fonction de V
0
et des
param` etres du c able. Travailler en coordonn ees cylindriques.
3. D eterminer lexpression et la valeur num erique de la puissance moyenne transport ee
par le signal, (i), en utilisant le vecteur de Poynting, (ii), en utilisant la th eorie des
lignes de transmission (pour rappel, Z
0
= (/2)ln[b/a]).
ELEN0076 11 - 8
Propagation selon z, sym etrie selon

E(r, z) =

E(r)e
z

H(r, z) =

H(r)e
z
Mode TEM :

E
z
=

H
z
= 0, pas de fr equence de coupure : = j = j

Dans un plan transverse ` a la direction de propagation, c- ` a-d z x e, les champs


pr esentent une forme semblable ` a celle des champs statiques

E(r) =

E
r
(r) r

H(r) =

H

(r)

1. Lignes de champ :

J
S
= n

H
-
-
-
-
-
-
-
-
-
-
-
-
-
-
-
-
-
-
+
+
+
+
+
+
+
+
+

S
= n

E
-
-
ELEN0076 11 - 9
2. Expressions des phaseurs

E et

H
Dans une section transverse, on d enit :
la tension :

V (z) =
_
b
a

E(z)

d et le courant :

I(z) =
_
C

H(z)

d
!! Les contours dint egration sont enti ` erement contenus dans un plan transverse
par application du th eor ` eme de Gauss sur une surface cylindrique de rayon r, on
d erive :

E(r) =

A
r
r et en z = 0 (pris au g en erateur)

V
0
=
_
b
a

A
r
r rdr =

Aln b/a

A =

V
0
lnb/a
Expression du champ electrique :

E(r, z) =

V
0
lnb/a
1
r
e
jz
r

E(r, z, t) =
V
0
ln b/a
1
r
cos(t z) r
ELEN0076 11 - 10
De


E = j

H on tire :


E
r
z
= j

H

E
r
=

H

avec =
_

Expression du champ magn etique:

H(r, z) =

V
0
ln b/a
1
r
e
jz

H(r, z, t) =
V
0
lnb/a
1
r
cos(t z)

ELEN0076 11 - 11
3. Puissance moyenne transport ee
a. A partir du vecteur de Poynting
<

S > =
1
2
Re(

)
=
V
2
0
2(ln b/a)
2
1
r
2
z
Puissance moyenne transport ee :
P =
_
S
<

S >

dS
=

V
2
0
ln b/a
= 2.4 kW
z

H
ELEN0076 11 - 12
b. A partir de la th eorie des lignes de transmission
P =
1
2
Re(

V (z)

(z)) =
1
2
[

V (z)[
2
Z
0
Z
0
=

V (z)

I(z)
est limp edance caract eristique de la ligne
Expressions des tension et courant :

V (z) =
_
b
a

E

d =

V
0
e
jz

I(z) =
_

H

d =
2
ln b/a

V
0
e
jz
Z
0
=

2
ln b/a = 52
Finalement
P =
V
2
0
2Z
0
=

V
2
0
lnb/a
= 2.4 kW
ELEN0076 11 - 13
8.2 Modes TM des guides dondes rectangulaires
On consid` ere un guide dondes m etallique de section rectangulaire. Ses parois sont des
conducteurs parfaits, lint erieur est rempli dun isolant de permittivit e electrique et de
perm eabilit e magn etique .
D eterminez les expressions de la composante

E
z
du champ electrique correspondant
aux modes TM, cest- ` a-dire les modes tels que

H
z
= 0.
Remarques :
Travaillez dans le domaine fr equentiel et limitez-vous ` a des solutions dondes
progressives, du type

E
z
(x, y, z) =

E
z
e
z
, o` u est la constante de propagation
(complexe).
ELEN0076 11 - 14
Equation donde pour la composante

E
z
(

2
x
2
+

2
y
2
+k
2
c
)

E
z
= 0 avec k
2
c
=
2
+
2

R esolution par la m ethode de s eparation de variables


On suppose

E
z
= f(x)g(y) :
g(y)

2
f(x)
x
2
+f(x)

2
g(y)
y
2
+k
2
c
f(x)g(y) = 0
1
f(x)

2
f(x)
x
2
+
1
g(y)

2
g(y)
y
2
+k
2
c
= 0
1
f(x)

2
f(x)
x
2
ind ep. de y et
1
g(y)

2
g(y)
y
2
ind ep. de x

1
f(x)

2
f(x)
x
2
= k
2
x
1
g(y)

2
g(y)
y
2
= k
2
y
avec k
2
c
= k
2
x
+k
2
y
Constantes n egatives solutions harmoniques
ELEN0076 11 - 15
f(x) = A
1
cos(k
x
x) +A
2
sin(k
x
x)
g(y) = A
3
cos(k
y
y) +A
4
sin(k
y
y)
Forme g en erale de

E
z
:

E
z
= (A
1
cos(k
x
x) +A
2
sin(k
x
x))(A
3
cos(k
y
y) +A
4
sin(k
y
y))e
z
C.L. sur les parois :

E
z
= 0 pour
_

_
x = 0, y = 0 A
1
= A
3
= 0
x = a k
x
=
m
a
, m = 1, 2, . . .
y = b k
y
=
n
b
, n = 1, 2, . . .

E
z
= E
0
sin(
mx
a
) sin(
ny
b
)e
z
ELEN0076 11 - 16
Les composantes tranverses des champs se d eduisent des equations en rotationnel.
Recherche de

H
y
(2)

H
y
=
1
j
_


E
x



E
z
x
_
(4)

E
x
=
1
j
_


H
y
+


H
z
y
_

H
y
=
1
k
2
+
2
_


H
z
y
+j


E
z
x
_
=
j
k
2
+
2


E
z
x
ELEN0076 11 - 17
Pour les autres composantes, les relations g en erales sont :

E
x
=
1
k
2
+
2
_


E
z
x
+j


H
z
y
_

E
y
=
1
k
2
+
2
_


E
z
y
+j


H
z
x
_

H
x
=
1
k
2
+
2
_


H
z
x
+j


E
z
y
_
ELEN0076 11 - 18
Modes TM
Forme des champs :

E
z
= E
0
sin(
mx
a
) sin(
ny
b
)e
z

H
z
= 0

E
x
=

k
2
+
2
m
a
E
0
cos(
mx
a
) sin(
ny
b
)e
z

E
y
=

k
2
+
2
n
b
E
0
sin(
mx
a
) cos(
ny
b
)e
z

H
x
=
j
k
2
+
2
n
b
E
0
sin(
mx
a
) cos(
ny
b
)e
z

H
y
=
j
k
2
+
2
m
a
E
0
cos(
mx
a
) sin(
ny
b
)e
z
m = 1, 2, . . . , n = 1, 2, . . .
ELEN0076 11 - 19
Caract eristiques de la propagation
Condition de propagation :

2
= k
2
c
k
2
=
_
m
a
_
2
+
_
n
b
_
2

2
=
1
v
2
(
2
c

2
) < 0
>
c
Pulsation, fr equence et longueur donde de coupure :

c
= v
_
_
m
a
_
2
+
_
n
b
_
2
f
c
=
v
2
_
_
m
a
_
2
+
_
n
b
_
2

c
=
2
_
_
m
a
_
2
+
_
n
b
_
2
Vecteur donde :
=

v

1
_
f
c
f
_
2
ELEN0076 11 - 20
Vitesse de phase et vitesse de groupe :
v
p
=

=
v
_
1
_
f
c
f
_
2
v
g
=
d
d
= v

1
_
f
c
f
_
2
v
p
v
g
= v
2
Longueur donde dans le G.O.

g
=
2

=

_
1
_
f
c
f
_
2
ELEN0076 11 - 21
Modes TE
Forme des champs :

H
z
= H
0
cos(
mx
a
) cos(
ny
b
)e
z

E
z
= 0

E
x
=
j
k
2
+
2
n
b
H
0
cos(
mx
a
) sin(
ny
b
)e
z

E
y
=
j
k
2
+
2
m
a
H
0
sin(
mx
a
) cos(
ny
b
)e
z

H
x
=

k
2
+
2
m
a
H
0
sin(
mx
a
) cos(
ny
b
)e
z

H
y
=

k
2
+
2
n
b
H
0
cos(
mx
a
) sin(
ny
b
)e
z
m = 0, 1, 2, . . . , n = 0, 1, 2, . . . !! m, n non nuls simultan ement
pas de mode TEM !!
ELEN0076 11 - 22
Mode fondamental TE
10
Mode dont la fr equence de coupure est la plus basse.
f
c
=
v
2a

c
= 2a =
_

2

_

a
_
2
Champs :

E
y
= E
0
sin
_
x
a
_
e
jz

H
x
=

E
0
sin
_
x
a
_
e
jz

H
z
= j
/a

E
0
cos
_
x
a
_
e
jz
ELEN0076 11 - 23
8.4 Mode dun guide dondes de section rectangulaire
On consid` ere un guide dondes de section rectangulaire, avec a = 1.5 cm et
b = 0.8 cm, et dont lespace entre les conducteurs est rempli par un di electrique de
caract eristiques = 0 ,
r
= 1 et
r
= 4.
On rel ` eve
H
x
= 2 sin
_
x
a
_
cos
_
3y
b
_
cos
_
10
11
t z
_
[A/m] .
D eterminer
1. le mode en op eration,
2. la fr equence de coupure correspondante,
3. le vecteur donde ,
4. la constante de propagation .
ELEN0076 11 - 24
1. Mode TE
13
et/ou TM
13
2. M eme fr equence de coupure pour les 2 modes
f
c
=
v
2

a
_
2
+
_
3
b
_
2
= 28.6 GHz avec v =
c

r
= 1.5 10
8
m/s
3. Vecteur donde
=

v

1
_
f
c
f
_
2
= 1719 rad/m
4. Constante de propagation
= j = j1719 m
1
ELEN0076 11 - 25
8.5 Perturbation des modes dun guide dondes de section rectangulaire
On ins` ere une plaque m etallique d epaisseur n egligeable dans le plan m edian dun guide
donde rectangulaire ` a air (la plaque est positionn ee en y = b/2).
Pour lesquels des modes suivants la r epartition des champs electromagn etiques est-elle
affect ee par linsertion de la plaque ?
modes TE
20
, TE
01
, TM
11
, TM
12
, TE
11
.
Justiez.
ELEN0076 11 - 26
Pas dalt eration des champs si en tout point de la plaque conductrice, les conditions
aux limites sont satisfaites, soit :
le champ electrique

E est perpendiculaire ` a la plaque :

E
x
=

E
z
= 0 pour y =
b
2
le champ magn etique

H est tangent ` a la plaque :

H
y
= 0 pour y =
b
2
Les modes non alt er es sont les modes : TE
20
, TM
12
ELEN0076 11 - 27
.tr,
7.
t' x u.
ot'
ce
+.
i i
7
//u, /t oj;
7t)o
aF-
| 1r
Tf?,,
T,
?'-o
-
*
=o
,4q2o
-
.:)
h
A>
-.
/1 /r aa
/
-
d
,qr
=
/f
'/"i
@
dl
/'ofrreh.
=He
t,tra
v
(
tu
6,
--
/ 4at
ry*o fu;o
Hx -'o
I
'
HJ-'
-)
a F//r?.*,
HJ=o
* s
4ccry
4
Hx?
/(
,.1
7
q
4-.
o lal7 o.H
4ey*o
?
'i
lt 2e^% g
cJ,
ry
H3
-r
/(c"%
,nl {,o
HJ:
,L,
'J
:'
Ht,'
J?o
x
--
/(ca4 y'"\U
ao
Ho
-' k e- lttrfl
(/Ab
,%
(hry
a
lr,2^
4
<
4z,U
4
cary
b
c4
b
C., A?
b
-, 4
/ieor/^.
/lJ eo ,j,
r
--
/(
sa@ Zc;?ttV
i
O
ab
/-/r: /r aary c2, tu
*<>
+c
q'zc;
fl,2
+
o
b
H,)' Ho t'f
o,ae;% zi'q!,
=o
qb
A"
s
/f zory ca? z?
Jqb'
H,: te.rq /"i
zo.? ic
/4r
-:)
a
freha-4;^
ELECTROMAGNETISME
Formulaire
1
dexercices
1 Rappels
1.1 Trigonom etrie
sin(a +b) = sin a cos b + cos a sin b
cos(a +b) = cos a cos b sin a sin b
sin a sin b =
1
2
cos(a b)
1
2
cos(a +b)
cos a cos b =
1
2
cos(a b) +
1
2
cos(a +b)
sin a cos b =
1
2
sin(a +b) +
1
2
sin(a b)
cos a sin b =
1
2
sin(a +b)
1
2
sin(a b)
1.2 Analyse vectorielle

( +) =

+

A +

B) =



A+



B

A +

B) =



A +



B

() =

+

(

A) =

A

+

A

B) =

B



A

A



B

A

B) =

A



B

B



A+ (

B

)

A (

A

)



=
2



A) = 0



= 0



A) =

(



A)

2

A

A

B) = (

A

)

B + (

B

)

A +

A (



B) +

B (



A)

A (

B

C) =

B (

C

A) =

C (

A

B)

A (

B

C) =

B(

A

C)

C (

A

B)
1. Version 31 mai 2012
1
1
.
3
C
h
a
n
g
e
m
e
n
t
d
e
c
o
o
r
d
o
n
n
e
e
s
Coordonn ees
cart esiennes cylindriques sph eriques

V
V
x
x +
V
y
y +
V
z
z
V
r
r +
1
r
V

+
V
z
z
V
r
r +
1
r
V

+
1
r sin
V



A
A
x
x
+
A
y
y
+
A
z
z
1
r

r
(rA
r
) +
1
r
A

+
A
z
z
1
r
2

r
(r
2
A
r
) +
1
r sin

(sin A

) +
1
r sin
A



A (
A
z
y

A
y
z
) x+ (
1
r
A
z

z
) r+
1
r sin
_

(sin A

)
A

_
r+
(
A
x
z

A
z
x
) y+ (
A
r
z

A
z
r
)

+
1
r
_
1
sin
A
r



r
(rA

)
_

+
(
A
y
x

A
x
y
) z
1
r
_

r
(rA

)
A
r

_
z
1
r
_

r
(rA

)
A
r

2
V

2
V
x
2
+

2
V
y
2
+

2
V
z
2
1
r

r
(r
V
r
) +
1
r
2

2
V

2
+

2
V
z
2
1
r
2

r
(r
2
V
r
) +
1
r
2
sin

(sin
V

) +
1
r
2
sin

2
V

dl dx x +dy y +dz z dr r +rd



+dz z dr r +rd

+r sin d

dS dy dz x +dxdz y +dxdy z r d dz r +dr dz



+r dr d z r
2
sin d d r +r sin dr d

+r dr d

d
3
r dxdy dz r dr d dz r
2
sin dr d d
2
2 Electrostatique
Dans le vide :
champ electrique produit par une charge ponctuelle :

E(P) =
q
4
0
r
2
r
q
p
r
r
^

0
=
1
36
10
9
F/m
- pour une distribution volumique de charges : application de la superposition

E(r) =
1
4
0
_
V
(r

)
|r r

|
3
_
r r

_
d
3
r

0
r
P
r

eqs. de Maxwell :

t
= 0 =



E = 0



E =

0
potentiel electrique :



E = 0 =

E =

V
potentiel electrique cr e e par une charge ponctuelle :
V =
q
4
0
r
+C
potentiel electrique cr e e par une distribution volumique de charges :
V (r) =
1
4
0
_
V
(r

)
|r r

|
d
3
r

+C
diff erence de potentiel :
V
A
V
B
=
_
A
B

E

dl
loi de Gauss :
_
S

E

dS =
Q

0
=
_
V

0
d
3
r
Q
S
n
eq. de Poisson et de Laplace

2
V =

0
( eq. de Poisson)
= 0 si = 0 ( eq. de Laplace)
champ et potentiel cr e es par un conducteur lin eique inniment long :

E(P) =

L
2
0
r
r
V (P) =

L
2
0
ln
r
0
r
+ Cst

L
P
r
r
^
z
^
3
capacit e dun condensateur = ensemble de deux conducteurs portant des charges egales et op-
pos ees +Q, -Q et pr esentant une diff erence de potentiel V
1
V
2
:
C =
Q
V
1
V
2
3 Milieux di electriques - Polarisation
vecteur polarisation :

P =
0

E
induction electrique :

D =
0

E +

P =
0
(1 +
e
)

E =
0

r

E =

E
charges li ees induites par la polarisation :

pol
=

pol
=

P n



D =

(
0

E +

P) = (
libres
+
pol
)
pol
=
libres
permittivit e effective :
=

tot
=

j(

pol
)
admittance equivalente dun condensateur (en r egime harmonique) :
Y = jC +G
avec
C =

f(d)
G =

tot
f(d) = f(d) +

pol
f(d)
G
C
I
G
I
C
avec f(d), facteur x e par la g eom etrie du condensateur
angle de perte :
tg =
I
G
I
C
=

tot

pertes par conduction : caract eris ees par la conductance


G
c
= f(d)
pertes par hyst er esis di electrique : caract eris ees par la conductance
G
pol
=

pol
f(d)
4
4 Magn etostatique
induction magn etique cr e ee par un l conducteur parcouru par un courant I : loi de Bio-Savart

B
0
r
P
r

Id

B(r) =

4
_
Id

(r r

)
|r r

|
3
avec : perm eabilit e magn etique du milieu, dans le vide :
0
= 4 10
7
H/m.
champ magn etique :

H =

eqs. de Maxwell :

t
= 0 =



H =

J



B = 0
potentiel vecteur :



B = 0 =

B =



A
avec

A =

4
_
I

dl
|r r

|
champ magn etique cr e e par un conducteur lin eique inniment long :

H =
I
2r

H
r
r
^
z
^
th eor` eme de Stokes : pour tout contour ferm e C d elimitant une surface non ferm ee S :
_
C

H

dl =
_
S



H

dS =
_
S

J

dS =

I
ux dinduction magn etique ` a travers une surface S (non ferm ee) :
=
_
S

B

dS
inductance L :
` a partir du ux dinduction cr e e par une boucle de courant I d elimitant une surface S :
L =

I
` a partir de l energie magn etique emmagasin ee dans un volume V :
W
M
=
1
2
_
V

B

H d
3
r =
1
2
LI
2
5
5 Propagation et diffusion
Equation g en erale homog` ene des ondes

E et

H :

2
_

H
_


2
t
2
_

H
_


t
_

H
_

_
= 0
En r egime sinusodal :

2
_

H
_

_
+
2

H
_

_
j
_

H
_

_
= 0
Milieu bon conducteur : >> , diffusion

2
_

H
_

_
j
_

H
_

_
= 0
Milieu bon isolant : << , propagation

2
_

H
_

_
+
2

H
_

_
= 0
Puissance transport ee par une onde

E(t) ,

H(t) :
puissance traversant une surface S :
_
S
(

E

H) d

S
vecteur de Poynting = puissance transport ee par unit e de surface

S =

E

H
pour des champs sinusodaux (phaseurs) :

S =
1
2
Re{

}
= puissance moyenne transport ee par unit e de surface
6 Effet pelliculaire - diffusion de londe

E
profondeur de p en etration ou epaisseur de peau :
=
_
2

6
imp edance equivalente du conducteur plan : par unit e de longeur du conducteur et pour un conduc-
teur de largeur unitaire,
Z
s
=
1 +j

= R
s
+jL
s
imp edance equivalente du conducteur cylindrique de rayon a : par unit e de longueur du conducteur
approximation basses fr equences (a < ) :
R =
1
a
2

L =

8
approximation hautes fr equences (a > ) :
R = L =
1
2a
7 Lignes de transmission en r egime harmonique ou sinusodal
7.1 Solutions des equations des t el egraphistes :

V (z) =

V
+
e
z
+

V

e
z

I(z) =
1
Z
0
_

V
+
e
z

e
z
_
avec
constante de propagation =
_
(R +jL)(G +jC) = +j
imp edance caract eristique Z
0
=
_
(R +jL)
(G+jC)
vitesse de phase v
p
=

longueur donde =
2

=
v
p
f
7.2 Cas particuliers :
Z
0

Ligne id eale R = G = 0
_
L
C
0

LC
Ligne ` a faibles pertes G C, R L
_
L
C
1
2
_
GZ
0
+
R
Z
0
_

LC
7.3 Ligne ferm ee sur une imp edance de charge Z
L
:
Coefcient de r eexion ` a la charge =

V
+
=
Z
L
Z
0
Z
L
+Z
0
Imp edance dentr ee de la ligne Z
in
(z = ) = Z
0
Z
L
cosh() +Z
0
sinh()
Z
0
cosh() +Z
L
sinh()
Pour une ligne id eale : = j Z
in
(z = ) = Z
0
Z
L
+jZ
0
tan()
Z
0
+jZ
L
tan()
Cas particuliers (ligne id eale) :
Z
in
()
ligne adapt ee Z
L
= Z
0
Z
0
0
ligne court-circuit ee Z
L
= 0 jZ
0
tan() -1
ligne ouverte Z
L
= jZ
0
cot() 1
7
7.4 Lignes id eales : ondes stationnaires
Ligne ferm ee sur une imp edance de charge Z
L
, coefcient de r eexion :
= ||e
j
Aux points z
max
tels que :
z = z +

+ 2k
V = V
max
= |

V
+
|(1 + ||) I = I
min
=
1
Z
0
|

V
+
|(1 ||) Z
in
(z
max
) = Z
max
= SZ
0
Aux points z
min
tels que :
z = z +

+ (2k + 1)
V = V
min
= |

V
+
|(1 ||) I = I
max
=
1
Z
0
|

V
+
|(1 + ||) Z
in
(z
min
) =Z
min
=
Z
0
S
avec S le taux dondes stationnaires :
S =
V
max
V
min
=
I
max
I
min
=
1 + ||
1 ||
Allure de lamplitude de la tension |

V (z)| :
|

V (z)| = |

V
+
|
_
1 + ||
2
+ 2|| cos(2z +

)
Ondes purement stationnaires :
S = = || = 1 = Z
L
= 0 ou Z
L
= ou Z
L
= jX
L

V

I
Z
L
= 0 2j

V
+
sin(z)
2

V
+
Z
0
cos(z)
Z
L
= 2

V
+
cos(z)
2j

V
+
Z
0
sin(z)
8 Guides dondes rectangulaires
a
b
x
z
y
8
8.1 Forme des champs
Rappel : projection des eqs. de Maxwell en rotationnel sur les axes x, y, z (r egime harmonique) :

E = j

H


E
z
y



E
y
z
= j

H
x
(1)


E
x
z



E
z
x
= j

H
y
(2)


E
y
x



E
x
y
= j

H
z
(3)

H = j

E


H
z
y



H
y
z
= j

E
x
(4)


H
x
z



H
z
x
= j

E
y
(5)


H
y
x



H
x
y
= j

E
z
(6)
Propagation des champs selon z :
_
_
_
_
_
_

H
_
_
_
_
_
_
(x, y, z, t) =
_
_
_
_
_
_

H
_
_
_
_
_
_
(x, y) exp(jt) exp(z)
Equation donde :
(
2
t
+k
2
c
)
_
_
_
_
_
_

H
_
_
_
_
_
_
= 0
avec k
2
c
=
2
+
2
= k
2
+
2
et
2
t
=

2
x
2
+

2
y
2
Expressions des composantes

E
x
,

E
y
,

H
x
,

H
y
en fonction des composantes

E
z
et

H
z
:

E
x
=
j
k
2
c
_


Ez
x
+


Hz
y
_

H
x
=
j
k
2
c
_



Ez
y


Hz
x
_

E
y
=
j
k
2
c
_


Ez
y
+


Hz
x
_

H
y
=
j
k
2
c
_



Ez
x
+


Hz
y
_

2
t

E
z
= k
2
c

E
z

2
t

H
z
= k
2
c

H
z
o` u on a suppos e labsence de pertes, = j, et on a d eni
k
2
c
=
2
+k
2
= k
2

2
.
9
modes TM :

E
z
=

E
0
sin(
mx
a
) sin(
ny
b
) exp(z)

H
z
= 0

E
x
=

2
+k
2
m
a

E
0
cos(
mx
a
) sin(
ny
b
) exp(z)

E
y
=

2
+k
2
n
b

E
0
sin(
mx
a
) cos(
ny
b
) exp(z)

H
x
=
j

2
+k
2
n
b

E
0
sin(
mx
a
) cos(
ny
b
) exp(z)

H
y
=
j

2
+k
2
m
a

E
0
cos(
mx
a
) sin(
ny
b
) exp(z)
modes TE :

E
z
= 0

H
z
=

H
0
cos(
mx
a
) cos(
ny
b
) exp(z)

E
x
=
j

2
+k
2
n
b

H
0
cos(
mx
a
) sin(
ny
b
) exp(z)

E
y
=
j

2
+k
2
m
a

H
0
sin(
mx
a
) cos(
ny
b
) exp(z)

H
x
=

2
+k
2
m
a

H
0
sin(
mx
a
) cos(
ny
b
) exp(z)

H
y
=

2
+k
2
n
b

H
0
cos(
mx
a
) sin(
ny
b
) exp(z)
8.2 Caract eristiques de la propagation
Pour le mode TE
mn
ou TM
mn
:
Fr equence de coupure f
c
=
1
2

_
(
m
a
)
2
+ (
n
b
)
2
=
v
2
_
(
m
a
)
2
+ (
n
b
)
2
Longueur donde de coupure
c
=
2

(
m
a
)
2
+(
n
b
)
2
et pour f > f
c
:
Constante de phase =

j
=

_
1 (
fc
f
)
2
Vitesse de phase v
p
=

=
v

1(
fc
f
)
2
Vitesse de groupe v
g
=
d
d
= v
_
1 (
fc
f
)
2
et v
p
v
g
= v
2
Longueur donde dans le guide
g
=
v
p
f
=
2

1(
fc
f
)
2
Puissance moyenne transport ee

S
z
=
1
2
Re
_

E
x

H

y


E
y

H

x
_
P
moy
=
_
a
0
dx
_
b
0
dy

S
z

10
9 Antennes
Potentiels vecteur et scalaire :

B =

A

E =


A
t
V
_

1
c
2

2
t
2
_

A =

J
_

1
c
2

2
t
2
_
V =

Potentiels retard es :

A(r, t) =

4
_
d
3
r
1

J(r
1
, t
|rr
1
|
c
)
|r r
1
|
,
V (r, t) =
1
4
_
d
3
r
1
(r
1
, t
|rr
1
|
c
)
|r r
1
|
.
Doublet de Hertz :
- champs (phaseurs)

A(r) =

I
0
h
4r
e
jkr
z

H =

I
0
h
4
e
jkr
_
jk
r
+
1
r
2
_
sin

,

E =

E
r
r +

E

E
r
=

I
0
h
2
e
jkr
_

r
2
+
1
jr
3
_
cos ,

I
0
h
4
e
jkr
_
j
r
+

r
2
+
1
jr
3
_
sin ,
=
_

k =

- puissance moyenne rayonn ee

S
r
=

2
_
_
k

I
0

h
4r
_
_
2
sin
2

P =
4
3

_
_
k

I
0

h
4
_
_
2
Antennes lin eaires longues : champs rayonn es

E =
j
4r
e
jkr
sin U()

,

H =

avec U() =
_
h
2
h
1
dz

I(z

)e
jkz

cos
11
Antennes r eseaux et facteur de r eseau F(, ) :
Soient plusieurs antennes i, de facteurs de forme commun U() et de positions r
i
. Chaque antenne i
est aliment ee par un courant dont lamplitude est egale ` a I(z) et la phase vaut
i
.
Le champ rayonn e par le r eseau a la forme

E(r) =
j
4r
e
jkr
sin U()F(, )

F(, ) =

i
e
j
i
e
jkr
i
cos
i
o` u r est le vecteur position du point dobservation et cos
i
= cos(r, r
i
).
12

Вам также может понравиться